You are on page 1of 58

100 Most Probable

Questions for Prelims


2017(Current Affairs)

COMPILED BY :
INDIA BHAI
(ASHUTOSH GUPTA)
100 Most Probable Questions for Prelims 2017(Current Affairs)

Dear Friends!

Now you all are in the last phase of your preparation for the prelims. So why not
augment your preparation with final revision of important topics for current affairs. Current
affairs form a crucial part of preparation for prelims since past years where it had a weightage
of 10-20 %. Its only in CSE 2016 prelims, it formed the major chunk of the questions having
more than 40% weightage. So the importance of current affairs cannot be ruled out as it will be
the determining part of the prelims preparation. Lets supplement your revision with this small
booklet of current affairs comprising of 100 questions.

This booklet consists of questions based on current affairs, India Year Book, Economic Survey,
Budget, Government Schemes and others important and miscellaneous areas. Go through the
questions and explanations and see whether you are able to attempt those questions correctly.
If still you are not able to attempt those questions correctly, then it will show your weakness in
such topics of current affairs. Then without wasting much time, immediately look up those
topics from your current affairs books from where you had prepared or Google it. This will help
you to complete your topics immediately and will boost your confidence for the exam.

Make the best use of this remaining time, in the revision of what you had learned and to plug
those missing loopholes, if any, in your final prelims preparations.

All the very best for prelims examination! May your efforts give you fruitful success.

Thanking you

India Bhai
(Ashutosh Gupta)

Like my face book page: India Bhai Page 2


100 Most Probable Questions for Prelims 2017(Current Affairs)

1. Under which Act, Prime Minister, on 8th of Nov2016, declared that the existing Rs.
500 and Rs. 1000 notes will no more be a legal tender?
(A) RBI Act 1934
(B) Banking regulation Act 1949
(C) Payment and Settlement Systems Act 2007
(D) Does not require any statutory backing

2. Consider the following statements:


(i) Bank notes are legal tenders
(ii) Bank notes are unlimited legal tenders
(iii) Bank notes are guaranteed by the Central Government
(iv) Bank notes are guaranteed by the RBI
Select the correct answer using the code given below:
(A) (i) & (iii) only (B) (i) & (iv) only
(C) (i), (ii) & (iii) only (D) (ii) & (iv) only

3. Consider the following statements regarding "Exchange Traded Funds (ETF)":


(i) ETFs are traded like stocks and can be bought and sold throughout the day
(ii) ETFs can be used as a vehicle for disinvestment
(iii) ETFs offers the benefit of diversification of risks
Select the correct answer using the code given below:
(A) (i) only (B) (i) & (ii) only
(C) (ii) & (iii) only (D) All of the above

4. Headcount ratio is defined as:


(A) Ratio of population below the poverty line to above the poverty line.
(B) Ratio of population above the poverty line to below the poverty line.
(C) Proportion of population that lives below the poverty line.
(D) Proportion of population that lives above the poverty line.

5. Which of the following statements are true regarding the Primary Agriculture Credit Societies
(PACS):
(i) It operates at the village level
(ii) It provides short term credit
(iii) It provides direct credit to small and marginal farmers
(iv) Regulated by RBI.
Select the correct answer using the code given below:
(A) (i) & (ii) only (B) (ii) & (iii) only
(C) (i), (ii) & (iii) only (D) All of the above

6. The Child Labour (Prohibition and Regulation) Amendment


Act, 2016 provides for
1. Prohibition of employment of children below 14 years in all occupations or processes except
where child helps his family.

Like my face book page: India Bhai Page 3


100 Most Probable Questions for Prelims 2017(Current Affairs)

2. Prohibition of employment of adolescents in hazardous occupations


3. Resettlement and rehabilitation of children found in state of bonded labour
Select the correct answer using the codes below
(A) 1 only (B) 1 and 2 only
(C)) 2 and 3 only (D) 1, 2 and 3

7. Consider the following about Sustainable Development Goals (SDGs) adopted from 2015.
1. It will apply to all UN member states and not just those in the developing world.
2. SDGs consist of more goals but lesser number of targets as compared to MDGs.
Which of the above is/are correct?
(A) 1 only (B) 2 only
(C) Both 1 and 2 (D) Neither 1 and 2

8. Consider the following about Climate Vulnerable Forum (CVF).


1. It is a group of all island nations that are threatened with sea level rise.
2. The group opposes international mechanisms on climate change projects as it views them as
non-participatory and exploitative.
Which of the above is/are correct?
(A) 1 only (B) 2 only
(C) Both 1 and 2 (D) None

9. Export competition has been a long standing issue in the WTOs negotiations. It has been
one of the major outcomes of the Nairobi package. It is mainly related to
(A) Agricultural export subsidies and export credits
(B) Preferential rules of origin for LDCs
(C) Temporary export tariff hikes to contain BoP crises
(D) Trade Facilitation agreement (TFA)

10. Consider the following with reference to the National Defence Fund (NDF).
1. It is contributed jointly by the Central government and state governments.
2. It is used for the modernization of border infrastructure in sensitive regions.
3. It is housed with the Reserve Bank of India (RBI).
Select the correct answer using the codes below.
(A) 1 and 2 only (B) 2 only
(C) 3 only (D) None of the above

11. Consider the following important legal provisions of Indian Penal Code (IPC) often seen in
news. Match them correctly with their mandate.
1. Section 499A of IPC A. Seditious writing or speech
2. Section 124A of IPC B. Defamation
3. Section 295 of IPC C. Outraging Communal sentiments
Select the correct match from the codes below.
(A) 1-A, 2-B, 3-C (B) 1-B, 2-A, 3-C
(C) 1-C, 2-C, 3-B (D) 1-B, 2-C, 3-A

Like my face book page: India Bhai Page 4


100 Most Probable Questions for Prelims 2017(Current Affairs)

12. The Geostationary Operational Environmental Satellite-R (GOES-R) launched by NASA will
be instrumental in
1. Issuing timely space weather alerts
2. Augmenting satellite-based search and rescue network
Which of the above is/are correct?
(A) 1 only (B) 2 only
(C) Both 1 and 2 (D) None

13. WIFEX 2016-17 programme of the Union Ministry of Earth Sciences (MoES) intends to study
(A) Different physical and chemical features of Fog
(B) Impact of electro-magnetic radiation on mineral dispersion
(C) Effects of mineral exploration on geology of Arctic
(D) Mechanism of urban floods

14. The salient features of the new Foreign Trade Policy (FTP) 2015-2020 is\are
1. Prioritization of sectors having domestic manufacturing capabilities and potential export
demand
2. Promotion of Special Economic Zones (SEZs) to boost exports
3. Paperless working to facilitate ease of doing business in India
Select the correct answer using the codes below.
(A) 1 and 2 only (B) 3 only
(C) 1 and 3 only (D) 1, 2 and 3

15. With reference to the FAME India Scheme, consider the following statements:
1. It aims to support the hybrid/electric vehicles market development and its manufacturing
ecosystem.
2. It is a part of the National Electric Mobility Mission Plan.
3. Only 4 wheeler passenger vehicles are covered under the scheme.
Which of the statements given above is/are correct?
(A) 1 only (B) 1 and 2 only
(C) 2 and 3 only (D) 1, 2 and 3

16. SENSAGRI, recently seen in news, is a:


(A) Project to develop indigenous prototype for drone based crop and soil health monitoring
system.
(B) Standard set by Indian Agricultural Research Institute for agriculture crop produce in India.
(C) Figure indicating the relative prices of shares on the Bombay Stock Exchange of agriculture
related firms.
(D) Project for development of effective structural health monitoring methods.

17. Synlight, recently seen in news, is


(A) A Solar Power tree developed by Council of Scientific & Industrial Research (CSIR).

Like my face book page: India Bhai Page 5


100 Most Probable Questions for Prelims 2017(Current Affairs)

(B) A remote sensing device that uses light in the form of pulsed laser to measure distances to
the Earth, developed by NASA.
(C) The world's largest artificial sun developed by German Aerospace Centre.
(D) A light synchronization device to modulate the frequency of light, developed by Indian
Institute of Technology, Bombay.

18. In the context of Banking, which of the following best describes haircut?
(A) It is the difference between the loan amount and the actual value of the asset used as
collateral.
(B) It is the reserve requirement that the commercial banks are required to maintain in the
form of gold and government approved securities.
(C) It is a process whereby all new restructured loans are classified as bad loans.
(D) It is the downsizing of the banking staff by restructuring the processes and reducing the
employees.

19. Parliamentary forums serve as platforms for ministers and officials to discuss critical issues.
With respect to this, consider the following statements:
1. They are mentioned in the Rules of Conducting Business of both houses of parliament.
2. The Speaker of the Lok Sabha is the ex-officio President of all the forums.
3. The duration of the members of the forum is co-terminus with their membership in the
respective Houses.
Which of the statements given above is/are correct?
(A) 1, 2 and 3 (B) 1 and 2 only
(C) 3 only (D) 2 and 3 only

20. With reference to Unstructured Supplementary Service Data (USSD), consider the following
statements:
1. It is a session based service where texts and interactions are not stored on the mobile phone.
2. A mobile phone must have an active internet connection for using USSD based services.
3. It can be used to improve financial inclusion in the country.
Which of the statements given above is/are correct?
(A) 1 and 2 only (B) 1 and 3 only
(C) 3 only (D) 1, 2 and 3

21. Which of the following statement(s) is / are correct?

1. GoI launched the .in domain in Devanagari script.


2. It covers 22 languages mentioned in the Constitution of India.
(A)Only 1 (B) Only 2
(C) Both 1 and 2 (D) Neither 1 and 2

22. Which of the following statement(s) is / are correct?

Like my face book page: India Bhai Page 6


100 Most Probable Questions for Prelims 2017(Current Affairs)

1. An action Plan is drawn up for Swachh Bharat Mission to become


reality by 2019, 125th birth anniversary of Mahatma Gandhi.
2. The action plan calls for an Open Defecation Free (ODF) India in 3 years time.
3. Priority is being given to build toilets for boys and girls in all schools of the country and this is
envisaged to be completed in one year.

Choose the correct answer from following choices:


(A) Only 1 (B) Only 1 and 2
(C) Only 3 (D) None of the above

23. In Saansad Adarsh Gram Yojana, a MP takes up a village for development. In this scheme,
what award categories have been decided?
1.Best Adarsh Grams.
2. Best District Collectors
3. Best MPs
4. Best Practices
Choose the correct answer from below options:

(A) Only 1 and 3 (B) Only 1, 2 and 4


(C) Only 1, 3 and 4 (D)All of the above

24. Five schemes have been launched under Pandit Deendayal Upadhyay Shramev Jayate
Karyakaram. Which of the following come under those?

1. Revamped Rashtriya Swasthya Bima Yojana.


2. Apprentice Protsahan Yojana
3. Universal Account Number Scheme
4. Random Inspection Scheme
Choose the correct answer from the below options:

(A) Only 1 and 2 (B) 2, 3 and 4


(C) 1, 3 and 4 (D) All of the above

25. Which of the following are among the 9 pillars of the Digital India Program?

1. Broadband Highways
2. e-Governance
3. IT sector Jobs
4. Early Harvest Programmes
Choose the correct answer from the below options:

Like my face book page: India Bhai Page 7


100 Most Probable Questions for Prelims 2017(Current Affairs)

(A) Only 1, 2 and 3 (B) Only 2 and 3


(C) Only 4 (D)All of the above

26. Wildlife Institute of India has proposed a modified classification which divides the country in
how many biogeographic regions?
(A) 10 (B) 6
(C) 4 (D) 5

27. India is divided into how many river basins/group of river basins?
(A) 20 (B) 8
(C) 12 (D) 36

28. Which of the following ministry / ministries is / are working on the Action Plan for Namami
Gange?
1. Ministry of Shipping
2. Ministry of Water Resources, River Development & Ganga Rejuvenation
3. Ministry of Tourism
4. Ministry of Environment, Forests and Climate Change
5. Ministry of Drinking Water Supply and Sanitation

Choose the correct answer from the below options:

(A) Only 2 and 5 (B)Only 2, 4 and 5


(C)Only 2 (D)All of the above

29. Which of the following statement(s) is / are correct with regards to initiatives under
Namami Gange?
1. Nirmal Dhara - includes restoration and conservation of wetlands, efficient irrigation
methods and rational agricultural practices.
2. Aviral Dhara includes sewage treatment and managing industrial discharge.
Choose the correct answer from the below options:

(A) Only 1 (B) Only 2


(C) Both 1 and 2 (D) Neither 1 and 2

30. The National Industrial Corridor Development & Implementation Trust (NICDIT), which will
implement all the proposed industrial corridors will be under the administrative control of
(A) Department of Industrial Policy and Promotion (DIPP)
(B) Department of Economic Affairs (DEA), Ministry of Finance
(C) Ministry of Heavy Industries
(D) Ministry of Road Transport and Highways

Like my face book page: India Bhai Page 8


100 Most Probable Questions for Prelims 2017(Current Affairs)

31. Which of the following modes of cashless transactions have been developed by National
Payment Corporation of India?
1. RuPay Card
2. Immediate Payment Service (IMPS)
3. BHIM App
4. Real Time Gross settlement (RTGS)
Select the correct answer using the code given below.
(A) 2 and 3 only (B) 1, 2 and 3 only
(C) 1 and 4 only (D) 1, 2, 3 and 4

32. What is "Long March-3B", recently in the news?


(A) Satellite launch rocket developed by China.
(B) World's fastest bullet Train developed by Japan.
(C) Recently constructed underground tunnel in Alps Mountains.
(D) World's first supersonic drone developed by USA.

33. Global Peace Index is launched by which of the following?


(a) United Nations Human Rights Council
(b) Institute for Economics and Peace
(c) World Bank
(d) United Nation High Commissioner for Refugees

34. Which among the following are the components of Pradhan Mantri Krishi
Sinchai Yojana?
1. Accelerated Irrigation Benefits Programme
2. Har Khet Ko Pani
3. Watershed development
4. Per drop more crop
Select the correct answer using the code given below.
(A) 1, 3 and 4 only (B) 1 and 3 only
(C) 2 and 4 only (D) 1, 2, 3 and 4

35. Consider the following statements with reference to 'SABLA' scheme:


1. It aims at all-round development of adolescent girls of 11-18 years of age.
2. It is being implemented using the platform of Integrated Child Development Scheme (ICDS)
through Aanganwadi Centres.
3. It is an initiative of the Ministry of Women and Child Development.
Which of the statements given above is/are correct?
(A) 1 only (B) 1 and 2 only
(C) 2 and 3 only (D) 1, 2 and 3

36. Which of the following pairs of Departments and Ministries of the Government of India are
not correctly matched?
1. Department of Official Language - Ministry of Culture

Like my face book page: India Bhai Page 9


100 Most Probable Questions for Prelims 2017(Current Affairs)

2. Department of Space - Ministry of Science and Technology


3. Department of Disinvestment Ministry of Commerce and Industry
4. Department of Land Resources - Ministry of Agriculture
Select the correct answer using the code given below.
(A) 1 and 2 only (B) 3 and 4 only
(C) 1, 2 and 4 only (D) 1, 2, 3 and 4

37. Which of the following statements are true regarding the Foreign Currency Non Resident
(FCNR) Accounts:
(i) Resident Indians and NRI's both can open
(ii) Only Foreign currency can be deposited
(iii) Rupee can be deposited
(iv) Can have both demand and time deposits
Select the correct answer using the code given below:
(A) (i) & (ii) only (B) (ii) only
(C) (ii) & (iv) only (D) (i), (ii) & (iv) only

38. Consider the following statements regarding the "Zero Defect, Zero Effect (ZED)" scheme:
(i) The scheme is for MSME sector
(ii) It ensures producing high quality manufacturing products
(iii) It ensures minimal negative impact on environment.
(iv) It is a cornerstone of the flagship Make in India programme
Select the correct answer using the code given below:
(A) (i) & (iv) only (B) (ii) & (iii) only
(C) (i), (ii) & (iii) only (D) All of the above

39. Railway budget has been merged with the Union budget from FY 2017-18. Which of the
following statements were true before the merger?
(i) The budgetary support given by the Central government to the Railways was treated as loan
in perpetuity (Capital-at-Charge).
(ii) Ministry of Railway used to remit dividend to the Central government.
Select the correct answer using the code given below:
(A) (i) only (B) (ii) only
(C) Both (i) & (i) (D) Neither (i) nor (ii)

40. National Mineral Exploration Policy (NMEP) is stated as the game-changing reform major by
the government agencies. Which of the following statements about the objective of NMEP
is/are correct?
1. Competitive bidding of the mineral block through e-auction on revenue sharing model.
2. The state government will be responsible for identifying mineral blocks for bidding.
3. Aero-geophysical survey of the country.
4. Quality research in a public-private partnership.

Select the correct answer using the code given below.

Like my face book page: India Bhai Page 10


100 Most Probable Questions for Prelims 2017(Current Affairs)

(A) 1 and 2 only (B) 2 and 3 only


(C) 2, 3 and 4 only (D) 1, 2, 3 and 4

41. NASA has recently launched a Mission OSIRIS-Rex Mission. In this context, which of the
following statements is/are correct?
1. The aim of this mission is to study an asteroid and return with a sample.
2. The mission seeks answers to the questions that are central to the human experience: like
where did we come from and what is our destiny.
3. It is a mission to study outer planets of solar system.

Select the correct answer using the code given below.


(A) 1 and 2 only (B) 2 only
(C) 3 only (D) None

42. Consider the following items.


1. GholwadChickoo
2. UttarakhandTejpatta
3. Purandar fig
4. Sangli raisins
Which of these products has/have Geographical Indicator (GI) tag?

Select the correct answer using the codes given below.


(A) 1, 2, and 3 (B) 2 and 3
(C) 1 only (D) 1, 2, 3 and 4

43. With reference to the Bharat Interface for Money (BHIM) mobile application, which of the
following statements is/are correct?
1. It is developed by the National Payment Corporation of India.
2. It has two levels of authentication as compared to the normal net banking which has three
levels of authentication.
3. There is a limit of Rs 5,000 per transaction and Rs 10,000 within 24 hours in BHIM

Select the answer using the codes given below.


(A) 1 only (B) 2 only
(C) 1 and 3 (D) 1, 2, 3

44. With reference to the recently launched "SwasthyaRaksha Programme", which of the
following statements is/are correct?
1. It is implemented by the Union Ministry of Health and Family Welfare.
2. Its aim is to promote and create health awareness in rural areas by arranging various health
camps, SwasthyaRakshan OPDs etc.

Select the answer using the codes given below.

Like my face book page: India Bhai Page 11


100 Most Probable Questions for Prelims 2017(Current Affairs)

(A) 1 only (B) 2 only


(C) Both 1 and 2 (D) Neither 1 nor 2

45. With reference to the "Pratham" satellite, which of the following statements is/are correct?
1. It is operated by the Indian Institute of Technology, Bombay.
2. Its primary mission is to count electrons in the Earth's ionosphere.
Select the answer using the codes given below.
(A) 1 only (B) 2 only
(C) Both 1 and 2 (D)Neither 1 nor 2

46. With reference to the RESOURCESAT 2A satellite, which of the following statements is/are
correct?
1. It carries two Solid State Recorders with a capacity of 200 Giga Bits each.
2. It carries three payloads - a high-resolution Linear Imaging Self Scanner(LISS-4) camera, a
medium resolution (LISS-3) camera, a coarse resolution Advanced Wide Field Sensor (AwiFS)
camera.

Select the answer using the codes given below.


(A) 1 only (B) 2 only
(C) Both 1 and 2 (D) Neither 1 nor 2

47. With reference to the "DeendayalUpadhyay Gram JyotiYojana", which of the following
statements is /are correct?
1. It is a scheme implemented by the Union Ministry of Rural Development.
2. "Garv-II" app can be used to ensure transparency in the implementation
of rural electrification program.

Select the answer using the codes given below.


(A) 1 only (B) 2 only
(C) Both1 and 2 (D) Neither 1 nor 2

48. Consider the following pairs of the organization and the corresponding union ministry:
Organization - Ministry
1. Geological Survey of India - Ministry of Earth Sciences
2. Anthropological Survey of India - Ministry of Human Resource Development
3. Archaeological Survey of India - Ministry of Science and Technology

Which of the above pairs is/are correctly matched?


(A) 1 only (B) 2 and 3
(C) 1, 2, 3 (D) None of the above

49. Which of the following statement is not correct about 'Insolvency and Bankruptcy Code,
2016'?
(A) The Code separates commercial aspects of the insolvency proceedings from judicial aspects.

Like my face book page: India Bhai Page 12


100 Most Probable Questions for Prelims 2017(Current Affairs)

(B) The code is administered by the Ministry of Finance.


(C) The code proposes setting up a regulator to register and regulate the functioning of
insolvency professionals.
(D) The Code provides for enabling mechanism on the issue relating to cross-border insolvency.

50. The Unstructured Supplementary Service Data (USSD) is a new technology that is used to
send text between a mobile phone and an application program in the network. Which of the
following statements regarding the USSD is/are not correct?
1. It works only in feature phones but not in smartphones.
2. It works only in Global System for Mobile communication (GSM) phones.
3. The USSD is faster than SMS.

Select the correct answer using the code given below.


(A) 1 only (B) 1 and 2 only
(C) 1, 2 and 3 (D) None of the above

51. Consider the following pairs:


Scheme/Programme: Ministry
1. Unnat Bharat Abhiyan: Ministry of Human Resource Development
2. Pradhan Mantri Adarsh Gram Yojana: Ministry of Social Justice and Empowerment
3. Pradhan Mantri Surakshit Matritva Abhiyan: Ministry of Health and Family Welfare

Which of the above pairs is/are correctly matching?


Select the answer using the codes given below.
(A) 1 only (B) 2 only
(C) 3 only (D) 1, 2, 3

52. With reference to the Blockchain technology recently in the news, which of the following
are its benefits?
1. There is no need for a third party/central authority while making transactions.
2. Since transactions are made by authorized miners, there are no hacking threats.
3. It records and validates each and every transaction.
Select the correct answer using the code given below.
(A) 1 and 2 (B) 2 and 3
(C) 1 and 3 (D) 1, 2, and 3
53. Consider the following statements regarding the RashtriyaGokul mission:
1. The mission aims to conserve and develop indigenous bovine breeds.
2. It has a provision for issuing Animal Health cards.

Which of the statements given above is/are correct?


(A) 1 only (B) 2 only
(C) Both 1 and 2 (D) Neither 1 nor 2

54. Which of the following items are included under the budget head Capital receipts of India?

Like my face book page: India Bhai Page 13


100 Most Probable Questions for Prelims 2017(Current Affairs)

1. Loans raised by the Government from the public


2. Interest and dividend on investments made by the Government
3. Fees and other receipts for services rendered by the Government
4. Disinvestment receipts
5. Recoveries of loans from State and Union Territory Governments
Select the correct answer using the code given below.
(A) 2 and 3 only (B) 1, 2 and 4 only
(C) 1, 4 and 5 only (D) 1, 2, 3, 4 and 5

55. According to the Economic Survey, what is relevant to Universal Basic Income?
(A) JAM: Jan Dhan, Aadhar and Mobile (B) Centre-state negotiations for program cost sharing
(C) Both of the above (D)Neither of the above

56. Economic survey 2016-2017 has projected the growth of the Indian economy in 2017-18 to
what range?
(A) 6.5-7.5 percent (B) 6.25-7.25 percent
(C) 6.2-7.2 percent (D) 6.75-7.25 percent

57. According to the Economic Survey, what is India's rank in wind power installations?
(A) 3rd (B) 4th
(C) 5th (D) 6th

58. Consider the following statements regarding PARA (Public Sector Asset Rehabilitation
Agency):

1. It will be charged with working out the largest and most complex cases of stressed assets.
2. It could solve the coordination problem, since debts would be centralised in one agency.
3. It would purchase specified loans from banks and then work them out, either by converting
debt to equity and selling the stakes in auctions or by granting debt reduction.

Identify the correct statements

(A) 1 & 2 only (B) 2 & 3 only

(C) 1 and 3 (D) 1, 2 and 3

59. Economic Survey 2016-2017 is the first to use __________.


(A) Big Data (B) Analytics
(C) Econometrics (D) Statistics

Like my face book page: India Bhai Page 14


100 Most Probable Questions for Prelims 2017(Current Affairs)

60. New estimates of labour migration in India show it was more pronounced for _________
(A) Females (B) Males
(C) Elderly workers (D) None of the above

61. Article 293 (3) of the Constitution is tool in the hands of centre to check fiscal deterioration
on part of states. What is Article 293(3)?

(A) States must take consent of the Centre for additional borrowing

(B) States must reduce their fiscal expenditure on centres instructions

(C) Centre may introduce financial emergency in state to check deteriorating financial situation

(D) Centre may raise interest rates on loans given to states

62. According to economic survey, which of following approaches can be used for gradual
introduction of UBI in India?

1. Offering UBI as a choice to beneficiaries of existing programs


2. Offering UBI to Women
3. Introduction of UBI for certain vulnerable groups like widows, pregnant women, old etc.
4. Introduce UBI in urban areas first

Identify the correct statements

(A) 1 & 2 only (B) 2 & 3 only

(C) All are correct (D) None are correct

63. As per Economic survey, what is twin balance sheet problem?

(A) Overleveraged companies and bad-loan-encumbered banks

(B) Underleveraged companies and bad-loan-encumbered banks

(C) Underleveraged government and bad-loan-encumbered banks

(D) Overleveraged economy and bad-loan-encumbered banks

Like my face book page: India Bhai Page 15


100 Most Probable Questions for Prelims 2017(Current Affairs)

64. Which of following are reasons for Indias twin balance sheet problem?

1. Difficulty in obtaining land and environmental clearances


2. Dampening impact of Global financial crisis
3. Increase in the interest rates by RBI to curb inflation

Identify the correct statements

(A) 1 & 2 only (B) 2 & 3 only

(C) All are correct (D) None are correct

65. Consider the following statements regarding the convergence patterns between different
Indian states.

1. On the financial front, though there has been income convergence but consumption
divergence.
2. On health front, there has been convergence across both fronts i.e. life expectancy at birth and
infant mortality rate.

Identify the correct statements

(A) 1 only (B) 2 only

(C) Both 1 and 2 (D) Neither 1 and 2

66. Consider the following statements with respect to Redistributive resource transfers (RRT)
curse?

1. It means Higher the RRT, the slower is growth, and the smaller is the share of manufacturing in
GSDP. Thus, the lower are the tax revenues and quality of governance.
2. Evidence suggests that there might be RRT curse in India.

Identify the correct statements

(A) 1 only (B) 2 only

(C) Both 1 and 2 (D) Neither 1 and 2

67. Which of following can be reason for strong internal trade in India?

1. Area based exemptions

Like my face book page: India Bhai Page 16


100 Most Probable Questions for Prelims 2017(Current Affairs)

2. Indirect Tax Distortions


3. Constitutional provisions promoting free trade

Identify the correct statements

(A) 1 & 2 only (B) 2 & 3 only

(C) 1 & 3 only (D) 1, 2 and 3

68. Which of following are short term consequences of Demonetization move of government?

1. Decline in Interest rates


2. Increase in financial savings
3. Increase in Public sector wealth
4. Digitalization of economy
5. Job losses

Identify the correct statements

(A) 1, 2, 4 & 5 only (B) 2, 3 & 4 only

(C) 1, 2, 3 and 5 (D)1, 2, 3, 4 and 5

69. Which of following are likely long term consequences of demonetisation move of
government?

1. Interest rates could fall further.


2. Reduction in stock of black money
3. Continuation of digital revolution
4. Increase in financial savings to the extent cash deposit ratio falls permanently
5. Stronger economy with least corruption

Identify the correct statements

(A) 1,2,3 & 5 only (B) 1,3,4 & 5 only

(C) 1, 2, 4 and 5 (D) 1, 2, 3, 4, and 5

70. Indias INDC (Intended Nationally Determined Contributions) under Paris agreement are:

1. Lower the emissions intensity of GDP by 33 to 35 per cent by 2030 from 2005 levels.

Like my face book page: India Bhai Page 17


100 Most Probable Questions for Prelims 2017(Current Affairs)

2. To increase the share of non-fossil fuels based power generation capacity to 60 per cent of
installed electric power capacity by 2030.
3. To create an additional (cumulative) carbon sink of 2.53 Gt CO2e through additional forest and
tree cover by 2030.

Identify the correct statements

(A) 1 & 3 only (B) 2 & 3 only

(C) 1 and 2 only (D) 1, 2 and 3

71. Consider the following w.r.t Inflation scenario in India:

1. The average CPI inflation declined in 2015-16 from 2014-15.


2. Headline Inflation is measured by CPI in India
3. The average WPI became negative in 2015-16
4. Pulses continued to be the major contributor of food inflation
5. Core inflation excludes food and fuel group

Identify the correct statements

(A) 1, 2 and 4 (B) 1, 2, 3 & 4 only

(C) 1, 2, 3 and 5 (D) 1, 2, 3, 4, and 5

72. Arrange the following countries in decreasing order of exports made by India to them
(2016-17 till Nov.):

1. USA
2. UAE
3. Hongkong

(A) 1 2 3 (B) 2 3 1

(C) 2 1 3 (D)1 3 2

73. Arrange the following countries in decreasing order of imports by India from them (2016-17
till Nov.):

1. UAE
2. China

Like my face book page: India Bhai Page 18


100 Most Probable Questions for Prelims 2017(Current Affairs)

3. USA

(A) 1 2 3 (B) 2 3 1

(C) 2 1 3 (D) 1 3 2

74. Apart from Union Budget, Which of following other documents are presented in Parliament
as part of Union Budget?

1. Macro-Economic Framework Statement


2. Fiscal Policy Strategy Statement
3. Medium Term Fiscal Policy Statement
4. Medium Term Expenditure Framework Statement

Which of following statements are correct?

(A) 1 & 3 (B) 2 & 3

(C) 1, 2 and 3 (D) 1, 2, 3 and 4

75. Which of following statements is presented along with Union Budget as per FRBM Act,
2003?

1. Macro-Economic Framework Statement


2. Fiscal Policy Strategy Statement
3. Medium-Term Fiscal Policy Statement
4. Medium-Term Expenditure Framework Statement

Which of following statements are correct?

(A) 1, 2 & 3 (B) 2 & 3

(C) 2, 3 and 4 (D) 1, 2, 3, and 4

76. Consider the following regarding sources of Money as per Union Budget 2017-2018 ?

1. Top sources of money are Borrowings & other Liabilities and Corporation Tax.
2. Non tax revenue is more than tax revenue.

Which of following statements are correct?

Like my face book page: India Bhai Page 19


100 Most Probable Questions for Prelims 2017(Current Affairs)

(A) Only 1 (B) Only 2

(C)Both 1 & 2 (D) Neither 1 and 2

77. Arrange the following subsidies in decreasing order of financial burden as per Union Budget
2017-18

1. Food.
2. Petroleum.
3. Fertilizer.

Which of following statements are correct?

(A) 1, 3, 2 (B) 2, 3, 1

(C) 3, 1, 2 (D) 1, 2, 3

78. Consider the following regarding sources of Money as per Union Budget 2017-2018?

1. Top sources of money are Borrowings & other Liabilities and Corporation Tax.
2. Non tax revenue is more than tax revenue.

Which of following statements are correct?

(A) Only 1 (B) only 2

(C) Both 1 & 2 (D)Neither 1 and 2

79. Which of following regarding timelines of action plans to tackle different diseases is
incorrect?

Disease Timeline for Elimination

(A) Kala-Azar and Filariasis 2017

(B) Leprosy 2018

Like my face book page: India Bhai Page 20


100 Most Probable Questions for Prelims 2017(Current Affairs)

(C) Measles 2020

(D) TB 2030

80. In the light of Union Budget 2017, consider the following statements in respect of initiatives
taken for YOUTH

1. SWAYAM platform, leveraging IT, to be launched with at least 350 online courses. to enable
students to virtually attend courses taught by the best faculty.
2. National Testing Agency to be set-up as an autonomous and self-sustained premier testing
organization to conduct all entrance examinations for higher education institutions.
3. Pradhan Mantri Kaushal Kendras to be extended to more than 600 districts across the country.
4. Skill Acquisition and Knowledge Awareness for Livelihood Promotion programme (SANKALP) to
be launched to provide market relevant training to 3.5 crore youth
5. Next phase of Skill Strengthening for Industrial Value Enhancement (STRIVE) will also be
launched.

Which of following statements are correct?

(A) 1, 2, 4 and 5 (B) 1, 2, 3 and 5

(C) 2, 3, 4 and 5 (D) 1, 2, 3, 4 and 5

81. In the light of Union Budget 2017, consider the following statements in respect of initiatives
taken for INFRASTRUCTURE

1. Unmanned level crossings on Broad Gauge lines will be eliminated by 2020.


2. By the end of 2017-18, high speed broadband connectivity on optical fibre will be available in
more than 1,50,000 gram panchayats, under BharatNet.
3. A DigiGaon initiative will be launched to provide tele-medicine, education and skills through
digital technology
4. A new and restructured Central scheme with a focus on export infrastructure, namely, Trade
Infrastructure for Export Scheme (TIES) will be launched in 2017-18

Which of following statements are correct?

(A) 1, 2 and 3 (B) 2, 3 and 4

(C) 1, 2 & 4 (D) 1, 2, 3 and 4

Like my face book page: India Bhai Page 21


100 Most Probable Questions for Prelims 2017(Current Affairs)

82. In the light of Union Budget 2017, consider the following statements in respect of initiatives
taken for FINANCIAL SECTOR?

1. Foreign Investment Promotion Board to be re-introduced in 2017-18.


2. A Computer Emergency Response Team for our Financial Sector (CERT-Fin) will be established
3. In line with the Indradhanush roadmap, ` 10,000 crores for recapitalization of Banks provided
in 2017-18
4. Lending target under Pradhan Mantri Mudra Yojana to be set at ` 2.44 lakh crores. Priority will
be given to Dalits, Tribals, Backward Classes and Women.

Which of following statements are correct?

(A) 1, 2 & 4 (B) 2, 3 & 4

(C) 1, 2 & 3 (D) 1, 2, 3 and 4

83. Which of the following statement / s is / are correct with regards to Clean Ganga Fund?

1. Domestic donors to the fund shall be eligible for tax benefit as applicable in the case of
Swachh Bharat Kosh.

2. The Fund will be managed by a trust headed by Minister for Water Resources.
Choose the correct answer from options below:

(A) Only 1 (B) Only 2


(C) Both 1 and 2 (D)Neither 1 and 2

84. Which of the following statement / s is / are correct with regards to National Mission for
Clean Ganga (NMCG)?

1. The area of operation of NMCG shall be Ganga River Basin including the states through which
Ganga flows as well as National Capital territory of Delhi.

2. The area of operation of NMCG may be extended to any extent as the National Ganga River
Basin Authority (NGRBA) may decide in states through which tributaries of Ganga flow.

Choose the correct answer from options below:

(A) Only 1 (B) Only 2


(C) Both 1 and 2 (D)Neither 1 and 2

Like my face book page: India Bhai Page 22


100 Most Probable Questions for Prelims 2017(Current Affairs)

85. Which scheme / schemes is / are fully supported by the Centre?

1. MGNREGA
2. Swachha Bharat Abhiyaan
3. National Rural Livelihood Mission (NRLM)
4. National Nutrition Mission
Choose the correct answer from options below:

(A) Only 1 and 4 (B)Only 1, 2 and 3


(C)Only 1 and 2 (D)All of the above

86. Dial. Gov is


(A) Mission to increase Tele-density in rural areas
(B)For information on eligibility of benefits under various social sector schemes
(C)Scheme for farmers to get information of markets
(D)Government helpline number for grievance redressal and lodging complaints

87. Ishan Uday is


(A) Special Package for MSME Industries in the Northeast
(B)Special Scholarship Scheme for North East Region (NER)
(C)Package for Sunrise Industries
(D) Exposure and internship programme for students of Northeastern colleges

88. What are objectives of Rashtriya Gokul Mission?

1) Development and conservation of indigenous breeds


2) Enhance milk production.
3) Distribute bulls for natural service.

Choose the correct answer from options below:

(A) Only 1 (B) Only 2


(C) 1 and 2 (D) 1, 2 and 3

89. Border Security Forces (BSF) role in war time is


1. Holding ground in assigned sectors
2. Aggression against invading enemy army
3. Guarding prisoners of war
4. Anti-infiltration duties; prevent unauthorized entry or exit from territory of India
5. Guide to army in border areas
Choose the correct answer from options below:
Like my face book page: India Bhai Page 23
100 Most Probable Questions for Prelims 2017(Current Affairs)

(A) Only 1, 2, 4 and 5

(B) Only 1, 3 and 5


(C) Only 1, 3, 4 and 5
(D)All of the above

90. The Access to Justice for Marginalized people project with UNDP support covered 7 states
in first phase. The project has also been approved for the period of five years (2012-2017) for
which states / regions?

1. Left-Wing Extremism affected states


2. North-Eastern States
3. Jammu and Kashmir
4. States with higher percentage of marginalized population than the national average
Choose the correct answer from options below:

(A) Only 1 and 2 (B) Only 2 and 3


(C) Only 4 (D) All of the above

91. The Hunar Zaika program is for

(A) Creating world class chefs to promote Indian food all over the world
(B) Skill development for creation and serving of high quality food in Trains and airplanes
(C) Upgrading hotels at important tourist places all over India.
(D)Upgrading the skills and hygiene standards of Street Food Vendors

92. A special initiative called Hunar Se Rozgar Tak (HSRT) and Hunar Se Rozgar Tak : Badhate
Kadam are launched by
(A) Ministry of Labour and Employment
(B) Ministry of Tourism
(C) Ministry of Human Resources
(D) Both A and C

93. Which of the following are initially identified circuits under SWADESH DARSHAN Scheme

1. Buddhist Circuit
2. North-East Circuit
3. Wildlife circuit
4. Spiritual Circuit
Choose the correct answer from options below:

Like my face book page: India Bhai Page 24


100 Most Probable Questions for Prelims 2017(Current Affairs)

(A) Only 1 and 2 (B) Only 1 and 4


(C)Only 1, 2 and 3 (D) All of the above

94. In India growth is mainly driven by


(A)Decrease of imports (B)Increase in exports
(C)Domestic demand (D)Controlling inflation

95. Madhukar Gupta Committee, recently seen in news, was constituted to recommend
measures for:
(A) encouraging deep sea fishing (B) strengthening border protection.
(C) broadening of tax base (D) preventing forest fires.

96. Which facility was announced for senior citizens in Budget 2017-2018?
(A) Aadhaar-enabled smart cards (B) BHIM app
(C) Shakti Kendras (D) None of the above

97. Indian Railways will focus on which of the following areas for the coming fiscal, as per the
Budget?
(A) Passenger Safety (B) Cleanliness
(C) Women Security (D) Only a and b

98. TSR Subramanian committee was constituted to review environmental regulation in India.
In this context, which of following is/are the recommendations of the committee?
1. Creation of an umbrella law by subsuming the existing environmental laws.
2. Establishment of National Environment Management Authority (NEMA) and State
Environment Management Authority (SEMA).
3. Establishment of Environment Reconstruction Fund.

Select the correct answer using the code given below.


(A) 1 only (B) 2 only
(C) 1 and 2 only (D) 1, 2 and 3

99. With reference to Electronic Voting Machines, consider the following statements:
1. The software used in these machines is installed into a onetime programmable chip to
prevent tampering.
2. All EVMs across India are networked through a private encrypted network of the Election
Commission of India.

Which of the statements given above is/are correct?


(A) 1 only (B) 2 only
(C) Both 1 and 2 (D) Neither 1 nor 2

Like my face book page: India Bhai Page 25


100 Most Probable Questions for Prelims 2017(Current Affairs)

100. The CEPI (Coalition for Epidemic Preparedness Innovations) has officially launched at 2017
World Economic Forum (WEF) to create new vaccines for emerging infectious diseases. The
CEPI has been formed with initial funding of $460 million from the governments/ organizations
of which of the following countries/organizations.

1. Germany

2. Japan

3. Norway

4. The Bill & Melinda Gates Foundation

5. Wellcome Trust.

Choose the correct options from the following.

(A) 1,2 and 3 (B) 1, 2, 3 and 4

(C) 1, 3, 4 and 5 (D) 1, 2, 3, 4 and 5

------------------------------------------------------------------

Like my face book page: India Bhai Page 26


100 Most Probable Questions for Prelims 2017(Current Affairs)

Answers with Explanations

1. Answer: A
Explanation:
As per the RBI Act 1934, Section 26, "on recommendation of the Central Board the Central
Government may, by notification in the Gazette of India, declare that, with effect from such
date as may be specified in the notification, any series of bank notes of any denomination shall
cease to be legal tender".

2. Answer: C
Explanation:
A country or its citizens may use many modes of exchange in their daily lives. History tells us
that ancient humans used salt and spices as currency. But Legal tender is the money that is
recognized by the law of the land, as valid for payment of debt. It must be accepted for
discharge of debt. The RBI Act of 1934, which gives the central bank the sole right to issue
bank/ urre otes, states that Ever a k ote shall e legal te der at a pla e i I dia i
pa e t for the a ou t e pressed therei .
Legal tender can be limited or unlimited in character. In India, coins function as limited legal
tender. Therefore, 50 paise coins can be offered as legal tender for dues up to Rs.10 and
smaller coins for dues up to Re1.. Currency notes are unlimited legal tender and can be offered
as payment for dues of any size.
As per the RBI Act 1934, all bank notes are guaranteed by the Central Government.

3. Answer: (d)
Explanation:
The concept of a mutual fund is that various investors/individuals put their money in a fund and
this fund is used to purchase shares/bonds of various companies thus diversifying the risk of
the investors. If the share price or bond price of the companies increase then the value of the
fund also increases and investors gain. If some individual wants to put money into the mutual
fund then it can be done only once after the market has closed for that given day.
Exchange Traded Funds (ETFs) are almost similar to that of mutual funds but they differ in the
sense that ETFs are traded on the stock exchange throughout the day. So if any investor wants
to purchase an ETF, they can always purchase it from the stock exchange/market throughout
the day like the shares of any company. The Central government has announced in the budget
that they will be creating an ETF of various central public sector companies so that to attract
investors to purchase ETFs, and through which the disinvestment can be done.

4. Answer: C
Explanation:
Headcount ratio is the proportion of population that lives below the poverty line. Hence as per
Rangarajan estimate, the poverty headcount ratio in 2011-12 was 29.5%.

5. Answer: C
Explanation:

Like my face book page: India Bhai Page 1


100 Most Probable Questions for Prelims 2017(Current Affairs)

Cooperative
Banks

Rural Urban
Cooperative Cooperative
Banks Banks

Short Term Long Term Single State Multi State

Primary
State District Central Agricultural
Cooperative Cooperative Credit
Banks Banks Societies
(PACS)

Primary Agricultural Credit Societies (PACS)


The rural co-operative credit system in India is primarily mandated to ensure flow of credit to
the agriculture sector. It comprises short-term and long-term co-operative credit structures.
The short-term co-operative credit structure operates with a three-tier system - Primary
Agricultural Credit Societies (PACS) at the village level, Central Cooperative Banks (CCBs) at
the district level and State Cooperative Banks (StCBs) at the State level.
A PACS is organized at the grass roots level of a village or a group of small villages. It is this
basic unit which deals directly with the rural (agricultural) borrowers, gives them loans and
collects repayments of loans given.
PACS are outside the purview of the Banking Regulation Act, 1949 and hence not regulated
by the Reserve Bank of India. SCBs/DCCBs are registered under the provisions of State
Cooperative Societies Act of the State concerned and are regulated by the Reserve Bank.
Powers have been delegated to National Bank for Agricultural and Rural Development
(NABARD) under Sec 35 A of the Banking Regulation Act to conduct inspection of State and
Central Cooperative Banks.

6. Answer: B
Explanation:
Like my face book page: India Bhai Page 2
100 Most Probable Questions for Prelims 2017(Current Affairs)

The Bill seeks to amend the Child Labour (Prohibition and Regulation) Act, 1986 to widen the
scope of the law against child labour and stricter punishments for violations.
Statement 2: It provides for addition of a new category of persons called adolescent . They are
person between 14 and 18 years of age. Their labour in hazardous industries is prohibited.
Government may confer powers on a District Magistrate (DM) to ensure that the provisions of
the law are properly carried out and implemented. It also empowers the government to make
periodic inspection of places at which employment of children and adolescents are prohibited.
Statement 3: There is no R&R provision in the bill presently. This matter is generally dealt by
various schemes and policies, such as National Child Policy and Bonded Labour rehabilitation
scheme (covered in previous tests).

7. Answer: A
Explanation:
Statement 1: It will apply to all UN member states and not just those in the developing world.
In this way, we hope they will become as universal as the Universal Declaration of Human
Rightsa vital element of the civic armoury in the fight for fairness.
Statement 2: SDGs consist of 17 goals and 169 targets in contrast to 8 goals and 18 targets in
MDGs set to expire in 2015. So, 2 is incorrect.
Goals are related to ending poverty; sustainable development; universal enrolment; improving
implementation etc. These goals are similar to MDGs, while others like reducing inequality
are new to SDGs. Instead of top-down approach in implementation of SDGs, bottom-up
approach is needed for the success of SDGs. You can see all goals under the link in Q Source.

8. Answer: D
Explanation: The CVF is also known as Vulnerable Twenty (V20) group.
Finance Ministers of the Vulnerable Twenty (V20) representing close to 700 million people
from 20 countries around the globe, hold their inaugural meeting during the World
Bank/International Monetary Fund Annual Meetings last year. So, 1 is incorrect.

The new group called for a significant mobilization of public and private finance for climate
action at the international, regional and domestic level ahead of the COP21 talks in Paris.
The V20 was created to share and scale up innovative approaches to climate finance developed
by those countries most affected by climate change. So, 2 is clearly incorrect.

9. Answer: A
Explanation:
While export subsidies for industrial products have been prohibited, such subsidies in
agricultural products were only subject to limited disciplines. So, the elimination of all forms of
agricultural export subsidies constituted one of the United Nations Sustainable Development
Goals launched in 2015. The proliferation of export subsidies in the years leading up to the
Uruguay Round was one of the key issues that were addressed in these negotiations
and they have remained a heated topic in recent years in the WTO So, the decision at the
recent Nairobi conference of WTO to fully eliminate any form of agricultural export subsidies is
an historic decision and constitutes a significant step in the reform of agricultural trade. It is

Like my face book page: India Bhai Page 3


100 Most Probable Questions for Prelims 2017(Current Affairs)

particularly meaningful for farmers in poor countries who cannot afford to compete with rich
countries which artificially boost their exports through subsidization.

10. Answer: C
Explanation:
Justification: The National Defence Fund was set up to take charge of voluntary donations in
cash and kind received for promotion of the national defence effort, and to decide on their
utilization.
Statement 1: The fund is entirely dependent on voluntary contributions from the public and
does not get any budgetary support. The fund accepts online contributions.
Statement 2: The Fund is used for the welfare of the members of the Armed Forces (including
Para Military Forces) and their dependents.
Statement 3: The Fund is administered by an Executive Committee, with PM as Chairperson,
and Defence, Finance and Home Ministers as Members. Finance Minister is the Treasurer of the
Fund and the Joint Secretary, PMO dealing with the subject is Secretary of the Executive
Committee. Accounts of the Fund are kept with the Reserve Bank of India

11. Answer:B
Explanation:
Section 295 of Indian Penal Code incriminates any act that outrages the religious
feelings or sentiments of others.
Section 124A criminalizes seditious acts, speech or writings.
Section 499A criminalizes defamation.
Section 66A (now repealed) criminalized a speech on electronic medium that has the
potential to hurt community sentiments.
These have been often seen in news due to many cases like Miss Ramya; JNSU President; Asim
Trivedi etc.

12. Answer: C
Explanation:
Statement 1: The satellite will boost the US s weather observation capabilities, leading to more
accurate and timely forecasts, watches and warnings. It is also crucial for the World in
identifying space and weather information in a substantially refined and modified format.
GOES-R is also first satellite to carry a lightning mapper instrument to geostationary orbit. It will
photograph lightning activity throughout Western Hemisphere about 200 times every second. It
will monitor the sun and relay crucial information to forecasters so they can issue space
weather alerts and warnings.
Statement 2: In all, data from GOES-R will result in new or improved meteorological, solar and
space weather products. It also will be part of the Search and Rescue Satellite Aided Tracking
(SARSAT) System, an international satellite-based search and rescue network operated by
NOAA.

13. Answer: A
Explanation:

Like my face book page: India Bhai Page 4


100 Most Probable Questions for Prelims 2017(Current Affairs)

It will study presence of extended periods of fog in northern parts of the country. It will be first
conducted at Indira Gandhi International Airport (IGIA), Delhi.
Fog is a visible mass consisting of cloud water droplets suspended in the air at or near
the Earth s surface.
In India, presence of heavy and extended period fog in the northern region (world s
most densely populated region) is one of the major weather hazards, impacting road,
aviation transportation, economy and public life.
Maximum fog occurrence is seen over the Northwest region.
In India, due to fog poor visibility (< 1 km) per year mostly occurs during the December-
February time period.
Recent studies conducted during the past 10-15 years on fog in India have prompted
significant socio-economic concern due to increase in frequency,
WIFEX aims to achieve better understanding of fog life cycle and ultimately improve
capability in fog prediction.
It will help to develop better now-casting (next 6 hours) and forecasting of winter fog
system on various time and spatial scales. It will also develop mechanism to reduce
adverse impact of Fog on aviation, transportation and economy, and loss of human life
due to accidents

14. Answer: D
Explanation:
It was announced with a vision to making India a significant participant in world trade by 2020
and to enable the country to assume a leadership position in the international trade discourse.
To increase India s exports of merchandise and services from USS 465.9 billIon in 201 3-
14 to USS 900 billion by 2019-20 and to raise India s share in world exports from 2 per
cent to 3.5 per cent during this period.
A new scheme, the Merchandise Exports from India Scheme (MEIS) aimed at me
simplification of the incentive structure and replacing multiple schemes is in place.
A new scheme, the Services Exports from India Scheme (SE IS) with expanded scope Is In
place which will benefit all service providers located in India instead of only Indian
service providers.
Allowing transferability and eater flexibility in using duty credit scrips which exporters
will get under MEIS and SEIS.
Envisages a move towards paperless working in a 24x7 environment to facilitate the
ease of doing business in.
Other sectors identified with great export potential include engineering, leather,
textiles, medical devices, natural resource based items, gems and jewellery, Agri
products and defence equipment.
Emphasis on the need to increase India s participation in global supply chains. Adoption
of the whole-of-government approach as a significant step towards better
coordination with state governments/other departments.

Like my face book page: India Bhai Page 5


100 Most Probable Questions for Prelims 2017(Current Affairs)

Indication of the revival or the interest subvention scheme for exports from identified
sectors for a period of three years, though sectoral coverage and rate of interest
subvention are not indicated.

15. Answer: B
Explanation:
Government of India launched the National Electric Mobility Mission Plan (NEMMP) 2020 in
2013. It aims to achieve national fuel security by promoting hybrid and electric vehicles in the
country. Government launched the Faster Adoption and Manufacturing of (Hybrid &) Electric
Vehicles (FAME India) under NEMMP 2020 in the Union Budget for 2015-16 with an initial
outlay of Rs. 75 Cr with the objective to support hybrid/electric vehicles market development
and Manufacturing eco-system. The scheme has 4 focus areas i.e. Technology development,
Demand Creation, Pilot Projects and Charging Infrastructure. Hence, statements 1 and 2 are
correct.
The FAME India Scheme is aimed at incentivising all vehicle segments i.e. 2 Wheeler, 3 Wheeler
Auto, Passenger 4 Wheeler Vehicle, Light Commercial Vehicles and Buses. Hence, statement 3
is not correct.
The scheme covers Hybrid & Electric technologies like Mild Hybrid, Strong Hybrid, Plug in
Hybrid & Battery Electric Vehicles. Recently, the government has withdrawn incentives given to
mild hybrid vehicles under FAME India.

16. Answer: A
Explanation:

The Indian Council of Agricultural Research (ICAR) through the Indian Agricultural
Research Institute (IARI) has formulated a collaborative research project entitled
SENSAGRI: SENsor based Smart AGRIculture involving six partner Institutes
(Agriculture & IT) to be funded by Information Technology Research Academy (ITRA),
Department of Electronics and Information Technology (DEITY), Ministry of
Communication and Information Technology (MCIT), Govt. of India and ICAR.
The major objective is to develop indigenous prototype for drone based crop and soil
health monitoring system using hyperspectral remote sensing (HRS) sensors. This
technology could also be integrated with satellite-based technologies for large scale
applications.
Drone technology based unmanned aerial vehicle (UAV) has ability for smooth scouting
over farm fields, gathering precise information and transmitting the data on real time
basis.
This capability could be used for the benefit of farming sector at regional/local scale for
assessing land and crop health; extent, type and severity of damage besides issuing
forewarning, post-event management and settlement of compensation under crop
insurance schemes.
17. Answer: C
Explanation:

Like my face book page: India Bhai Page 6


100 Most Probable Questions for Prelims 2017(Current Affairs)

The world s largest artificial sun, called Synlight, by the German Aerospace Centre (DLR)
was switched on the 23rd of March 2017, with the intention of innovating the
production of renewable energy.
Synlight is housed in a three-storey building in Jlich, Germany and is composed of 149
Xenon short-arc lamps which can produce a scorching heat of up to 3,000 degrees
Celsius.

18. Answer: A
Explanation:
To deal with mounting bad loans afflicting banks, the government is likely to come out
with a set of measures for faster resolution of non-performing assets. The policy may
contain the processes relating to haircut and one-time settlement to be carried out by
banks for faster resolution of high-value cases, adding that the big-loan defaults
constitute about 70% of the total non-performing assets.
A haircut is the difference between the loan amount and the actual value of the asset
used as collateral. It reflects the lender's perception of the risk of fall in the value of
assets. But in the context of loan recoveries, it is the difference between the actual dues
from a borrower and the amount he settles with the bank.
This is done because the lender gets at least some amount back instead of not getting
any money at all.
When collateral is being pledged, the degree of the haircut is determined by amount of
associated risk to the lender. These risks include any variables that may affect the value
of the collateral in the event that the lender has to sell the security due to a default by
the borrower. Variables that may influence that amount of a haircut include price, credit
and liquidity risks of the collateral.

19. Answer: C
Explanation:
Statement 1 is not correct : It is not mentioned in the Rules of conducting Business of
the Parliament. Parliamentary Forums provide a platform to the members to have
interactions with ministers, sensitise members about key areas and prepare data-base.
Each Forum consists of not more than 31 members (excluding the President and ex-
officio Vice Presidents) out of whom not more than 21 are from the Lok Sabha and not
more than 10 are from the Rajya Sabha. Members (other than the President and Vice-
Presidents) of these forums are nominated by the Speaker/Chairman from amongst the
leaders of various political parties/groups or their nominees, who have special
knowledge/keen interest in the subject.
Statement 2 is not correct :The Speaker of Lok Sabha is the President of all the Forums
except the Parliamentary Forum on Population and Public Health wherein the Chairman
of Rajya Sabha is the President and the Speaker is the Co-President. The Deputy
Chairman of Rajya Sabha, the Deputy Speaker of Lok Sabha, the concerned Ministers
and the Chairmen of Departmentally-Related Standing Committees are the ex-officio
Vice-Presidents of the respective Forums.

Like my face book page: India Bhai Page 7


100 Most Probable Questions for Prelims 2017(Current Affairs)

Statement 3 is correct : The duration of the office of members of the forum is co-
terminus with their membership in the respective Houses. A member may also resign
from the forum by writing to the Speaker/Chairman.

20. Answer: B
Explanation:
Statement 1 is correct: USSD is a session-based service which runs as a real-time open
session between the application and the end-user. While SMS is a store and forward
technology, USSD texts and interactions are not stored on the mobile phone. SMS
content remains stored in the mobile phone memory. Additionally, USSD can have up to
182 characters, while SMS is limited to 160 characters.
Statement 2 is not correct and statement 3 is correct: This service allows mobile
banking transactions using basic feature mobile phone, there is no need to have mobile
internet data facility for using USSD based mobile banking. It is envisioned to provide
financial deepening and inclusion of underbanked society in the mainstream banking
services. *99# service has been launched to take the banking services to every common
man across the country. Key services offered under *99# service include, interbank
account to account fund transfer, balance enquiry, mini statement besides host of other
services.

21. Answer: D: Neither 1 and 2


Explanation:
GoI launched the .bharat domain in Devanagari script covering 8 languages including Hindi,
Konkani and Marathi.
22. Answer: C: Only 3
Explanation:
Swachh Bharat is to be a reality by 2019, which is 150th anniversary of Mahatma Gandhi. ODF
target is in 5 years.
23. Answer: B: Only 1, 2 and 4
Explanation:
The four award categories are Best Adarsh Grams, Best District Collectors, Best Practices, and
Best Charge Officers.
24. Answer: D: All of the above
Explanation:
The five schemes are - Revamped Rashtriya Swasthya Bima Yojana, Apprentice Protsahan
Yojana, Universal Account Number Scheme, Random Inspection Scheme and Shram Suvidha
Portal
25. Answer: D: All of the above

Like my face book page: India Bhai Page 8


100 Most Probable Questions for Prelims 2017(Current Affairs)

Explanation:
The nine pillars are Broadband Highways, Universal Access to Mobile Connectivity, Public
Internet Access Programme, e-Governance Reforming Government through Technology, e-
Kranti (NeGP 2.0) Electronic delivery of services, Information for All, Electronics
Manufacturing Target NET ZERO Imports, IT for Jobs, Early Harvest Programmes.
26. Answer: A: 10
Explanation:
They are Trans-Himalayan; Himalayan; Indian Desert; Semi-Arid; Western Ghats; Deccan
Peninsula; Gangetic Plain; North East India; Islands; and Coasts.
27. Answer: A: 20
Explanation:
Out of those 20, 12 are major river basins and 8 are composite river basins.
28. ANSWER: D: All of the above
Explanation:
The entire above plus Ministry of Urban Development and Ministry of Rural development
29.Answer: D : Neither 1 and 2
Explanation:
Nirmal Dhara takes up issue of sewage and Aviral Dhara focuses on restoration and
conservation of wetlands, efficient irrigation methods and rational agricultural practices. There
will also be promotion of Shipping and Tourism.

30. Answer: A
Explanation:
The Union Government has approved the re-designation of the Delhi-Mumbai Industrial
Corridor Project Implementation Trust Fund (DMICPTF) as National Industrial Corridor
Development & Implementation Trust (NICDIT).
NICDIT will be the apex body to oversee integrated development of all industrial
corridors across the country.
NICDIT will function under the administrative control of Department of Industrial Policy
and Promotion (DIPP) i.e. Ministry of Commerce & Industry.
It will consist of Minister of Commerce & Industry, Minister of Railways, Minister of
Road Transport & Highways, Minister of Shipping, Vice-Chairman of NITI Aayog and
Chief Ministers of states concerned as members. NICDIT will implement all the five
proposed industrial corridors, together covering 15 States.

31. Answer: B
Explanation:

Like my face book page: India Bhai Page 9


100 Most Probable Questions for Prelims 2017(Current Affairs)

National Payment Corporation of India has developed RuPay Card, IMPS and BHIM App as e-
payment services. RTGS is not facilitated by NPCI. In RTGS, the settlements of funds transfer
happen through RBI.

32. Answer: A
Explanation:
Long March 3B is a Chinese orbital carrier rocket. It is mainly used to place
communications satellites into geosynchronous orbits.
China recently launched the first satellite dubbed as The Tiantong-01 for mobile
telecommunication. It was launched on-board of the Long March-3B carrier rocket from
Xichang Satellite Launch Centre.

33. Answer: B
The Global Peace Index (GPI) measures the relative position of nations' and regions'
peacefulness. It is launched by Institute for Economics and Peace (IEP). IEP develops it in
consultation with an international panel of peace experts from peace institutes and
think tanks with data collected and collated by the Economist Intelligence Unit.
India is ranked as the 141st country (out of 163 countries) in GPI 2016.
Iceland has been ranked as the most peaceful country followed by Denmark and Austria.
Syria has been named as the least peaceful preceded by South Sudan, Iraq, and
Afghanistan.

34. Answer: D
Explanation:
Pradhan Mantri Krishi Sinchai Yojana (PMKSY) conceived in 2015 is an umbrella scheme for
coverage of more and more area under assured irrigation as early as possible. Its major
components are:
(i) Accelerated Irrigation Benefits Programme (AIBP) for major and medium irrigation including
National projects;
(ii) Har Khet Ko Pani which includes command area development and water management
(CAD&WM) works, surface minor irrigation, irrigation through groundwater and repair,
renovation and restoration (RRR) of water bodies;
(iii) Per drop more crop for promotion of micro irrigation ; and
(iv) Watershed development for rain water harvesting, effective management of the run-off
water, prevention of soil erosion, regeneration of natural vegetation and recharging of the
ground water table.
Hence, all the statements are correct.

35. Answer: D
Explanation:
Statement 1 is correct: The Scheme aims at empowering adolescent girls of 11-18 years by
improvement of their nutritional and health status and upgrading various skills like home skills,
life skills and vocational skills. Thus the primary aim is health and nutrition of girls.

Like my face book page: India Bhai Page 10


100 Most Probable Questions for Prelims 2017(Current Affairs)

Statement 2 is correct: The monitoring and supervision mechanism set up under the
Integrated Child
Development Services (ICDS) Scheme is used for the scheme Sabla as well.
Statement 3 is correct: It is an initiative of the Ministry of Women and Child Development.

36. Answer: D
Explanation:
Department of Official Language - Ministry of Home Affairs.
Department of Pharmaceuticals - Ministry of Chemicals and Fertilizers.
Department of Disinvestment - Ministry of Finance.
Department of Food and Public Distribution - Ministry of Consumer Welfare, Food and
Public Distribution.
Department of Space is not under any ministry.
Department of Land Resources - Ministry of Rural Development

37. Answer: B
Explanation:
Foreign Currency Non Resident (FCNR) Account can be opened only by NRI's/PIO's. This account
can be maintained in any freely convertible foreign currency but only in the form of term
deposits. The interest and principal are non taxable and freely repatriable.

38. Answer: D
Explanation:
Micro Small and Medium Enterprises (MSMEs) sector is crucial for the economic progress of
India and it must match global quality control standards. The Zero Defect, Zero Effect (ZED)
scheme was launched in October 2016 to ensure that all the MSMEs are delivering top quality
product and using clean technology. This means the public will now onwards be able to use
clean technology products and they will also set parameters that are specific to each industry.
The main purpose of the scheme is to match the global quality control standards.
The ZED scheme is the cornerstone of the Make in India project which aims to turn India into a
manufacturing hub and generate jobs and increase incomes and boost the overall economy of
the country.

39. Answer: C
Explanation:
The budgetary support used to be provided (before the merger) to the Railways is basically the
Centre's investment in Railways and treated as loan in perpetuity (and is also called as
Capital-at-Charge) with a liability of dividend payment by the Railways to the Centre
representing interest on this loan. Capital at charge of Railways currently stands at nearly Rs.
3 lakh crore and annual dividend liability is around Rs. 10,000 crore. With merger of the
budget this liability was expected to vanish and Railways would have these resources for
investment (and this was one of the biggest arguments made in favour of scrapping the
Railway Budget). Hence, with the merger of the Railway budget, the Railways was expected to

Like my face book page: India Bhai Page 11


100 Most Probable Questions for Prelims 2017(Current Affairs)

get some fiscal benefits.


But after the presentation of the budget, the Finance ministry has asked the Railway ministry
to still remit the annual dividends it receives from the 14 Central Public Sector Units (CPSUs)
under its preview and it has become a bone of contention between the Finance Ministry and
the Railway Ministry.

40. Answer: D
Explanation:
NMEP is developed by the Mining Ministry and approved by the cabinet. It contains some major
policy changes such as -
Making public baseline geo-scientific data of world standards
Quality research in a public-private partnership
Special initiatives for search of deep-seated and concealed deposits,
Quick aero-geophysical surveys of the country,
Creation of a dedicated geo-science database
Establishment of a not-for-profit autonomous institution, National
Centre for Mineral Targeting (NCMT), to address the mineral exploration challenges.
Reimbursement of the exploration expenditure on the normative cost basis, if no
mineral reserves are found.

41. Answer: A
Explanation:
Learning Zone: The Origins, Spectral Interpretation, Resource Identification, Security, Regolith
Explorer (OSIRIS-REx) is a NASA asteroid study and sample return mission launched in
September 2016.OSIRIS-REx seeks answers to the questions that are central to the human
experience: Where did we come from? What is our destiny? Asteroids, the leftover debris from
the solar system formation process, can answer these questions and teach us about the history
of the sun and planets.
Mission Objectives: Return and analyze a sample of Bennu s surface(Bennu is the name of the
asteroid), Map the asteroid, Document the sample site, Measure the orbit deviation caused by
non-gravitational forces (the Yarkovsky effect), Compare observations at the asteroid to
ground-based observations

42. Answer: D
Explanation:
Learning Zone: Tejpata or Indian bay leaf, has become the first botanical plant from
Uttarakhand to receive the Geographical Indications (GI) tag. Figs from Saswad, Purandar, stand
out for its size, colour, taste and high iron content. Moreover, it grows on red soil, which adds
to the natural benefits.
The golden-green Sangli raisins melt in one's mouth and have fewer wrinkles on the outside,
which adds to its taste. GholwadChickoo belongs to the Dahanu-Gholvad belt of Palghar

43. Answer: C

Like my face book page: India Bhai Page 12


100 Most Probable Questions for Prelims 2017(Current Affairs)

Explanation:
BHIM is a digital payment solution app based on the Unified Payments Interface (UPI) from the
National Payments. Corporation of India (NPCI), the umbrella organisation for all retail
payments systems in India. BHIM also has options to transfer via IFSC and MMID as well for
non-UPI banks. It has a facility to store a limited amount of money in a mobile wallet like Paytm
or MobiKwik. It has three levels of authentification-one is the device ID and mobile number,
then the bank account which you are linking to this app, and the finally the UPI PIN which
is needed to complete the transaction
There is a limit of 10,000 per transaction and Rs 20,000 within 24 hours in BHIM.

44. Answer: B
Explanation:
The" SwasthyaRaksha Programme" is launched by the Ministry of AYUSH to promote health
and health education awareness in rural areas. SwasthyaRakshan OPDs AndSwasthyaParikshan
Camps are organised on weekly basis in villages. Several mass campaigns have been arranged
with a focus on environmental, social and personal hygiene.

45. Answer: C
Explanation:
Pratham is an Indian ionospheric research microsatellite which will be operated by the Indian
Institute of Technology Bombay as part of the Student Satellite Initiative. Its primary mission is
to count electrons in the Earth's ionosphere. It was successfully launched on 26 th September
2016 from Satish Dhawan Space Centre, Sriharikota, Andhra Pradesh along with 7 other
satellites on PSLV C-35.

46. Answer: C
Explanation:
RESOURCESAT-2A is a Remote Sensing satellite intended forresource monitoring.
RESOURCESAT-2A carries three payloads which are similar to those of RESOURCESAT-1
and RESOURCESAT-2.
They are a high-resolution Linear Imaging Self Scanner (LISS-4) camera operating in
three spectral bands in the Visible and Near Infrared Region (VNIR) with 5.8 m spatial
resolution and steerable up to 26 deg across the track to achieve a five-day revisit
capability.
The second payload is the medium resolution LISS-3 camera operating in three spectral
bands in VNIR and one in Short Wave Infrared (SWIR) band with 23.5 m spatial
resolution.
The third payload is a coarse resolution Advanced Wide Field Sensor (AWiFS) camera
operating in three spectral bands in VNIR and one band in SWIR with 56 m spatial
resolution.
RESOURCESAT-2A carries two Solid State Recorders with a capacity of 200 Giga Bits each to
store the images taken by its cameras which can be read out later to ground stations.

47. Answer: B
Like my face book page: India Bhai Page 13
100 Most Probable Questions for Prelims 2017(Current Affairs)

Explanation:
"DeenDayalUpadhyaya Gram JyotiYojana" is a Government of India scheme designed to provide
continuous power supply to rural India. It is implemented by the Ministry of Power. The
Ministry of Power has also launched a new app, "GARV-II" to provide real-time data of all six
lakh villages of the country. The app is envisaged to ensure transparency in the implementation
of rural electrification program.

48. Answer: D
Explanation:
Geological Survey of India, Anthropological Survey of India and Archaeological Survey of India
are independent organizations of the government of India.
Geological Survey of India is a part of the Ministry of Mines. Anthropological Survey of India and
Archaeological Survey of India are part of the Ministry of Culture.

49. Answer: B
Explanation:
Salient features of the Insolvency and Bankruptcy Code, 2016 are,
The Code separates commercial aspects of the insolvency proceedings from judicial
aspects. While Insolvency Professionals (IPs) will deal with commercial aspects, judicial
issues will be handled by proposed Adjudicating Authorities.
The Code provides a fast track insolvency resolution process.
The Code also addresses the important issue relating to cross-border insolvency by
providing the enabling mechanism on the subject.
The code proposes setting up a regulator to register and regulate the functioning of
insolvency professional agencies, insolvency professionals and information utilities.
The Insolvency and Bankruptcy code was spearheaded by Ministry of Finance. However, the
administration of the Insolvency and Bankruptcy Code, 2016 has been transferred to the
Ministry of Corporate Affairs w.e.f. 29th July 2016.

50. Answer: A
Explanation:
The Unstructured Supplementary Service Data (USSD), sometimes referred to as Quick Codes
or Feature codes , is a protocol used by GSM cellular telephones to communicate with the
service provider s computers. USSD can be used for WAP browsing, prepaid callback service,
mobile money services, location-based content services, menu-based information services, and
as part of configuring the phone on the network.
USSD messages are up to 182 alphanumeric characters long. Unlike Short Message Service
(SMS) messages, USSD messages create a real-time connection during a USSD session. The
connection remains open, allowing a two-way exchange of a sequence of data. This makes
USSD more responsive than services that use SMS.

51. Answer: D
Explanation:

Like my face book page: India Bhai Page 14


100 Most Probable Questions for Prelims 2017(Current Affairs)

Unnat Bharat Abhiyan is a Ministry of Human Resource Development, Government of India


programme to uplift rural India.The programme is being launched in collaboration with the
Indian Institutes of Technology (IITs), National Institutes of Technology (NITs) and other leading
Government Engineering Institutes like College of Engineering, Pune across the country.
Pradhan Mantri Adarsh Gram Yojana (PMAGY) is a rural development programme launched by
the Central government's under the Ministry of Social Justice and Empowerment in the
financial year 200910 for the development of villages having a higher ratio (over 50%) of
people belonging to the scheduled castes through convergence of central and state schemes
and allocating financial funding on a per village basis.
Pradhan Mantri Surakshit Matritva Yojana has been launched by PM Modi under the Ministry
of Health and family Welfare to provide healthcare facilities to pregnant women.

52. Answer: D
Explanation:
The Block chain is a new data structure that is secure, cryptography-based and distributed
across a network.
Benefits:
Records and validates each and every transaction and hence is secure and reliable. Transactions
are made by authorized miners and hence there are no hacking threats.
No need of a third party/central authority.
Decentralization of technology.

53. Answer: A
Explanation: With a view to conserve and develop indigenous bovine breeds, Rashtriya Gokul
Mission, a new initiative under National Programme for Bovine Breeding and Dairy
Development has been launched for the first time in the country. Animal Health card ( Nakul
Swasthya Patra ) is provided under Pashudhan Sanjivani, an animal Wellness Programme.

54. Answer: C
Explanation:
The capital receipts are loans raised by the Government from the public (these are termed as
market loans), borrowings by the Government from the Reserve Bank of India and other parties
through the sale of Treasury Bills, the loans received from foreign Governments and bodies,
disinvestment receipts and recoveries of loans from State and Union Territory Governments
and other parties.
Capital payments consist of capital expenditure on acquisition of assets like land, buildings,
machinery, equipment, as also investments in shares, etc., and loans and advances granted by
the Central Government to the State and the Union Territory Governments, Government
companies, Corporations and other parties.

55. Answer: C

Like my face book page: India Bhai Page 15


100 Most Probable Questions for Prelims 2017(Current Affairs)

Explanation:
The Economic Survey 2016-17 tabled in Parliament today by the Union Finance Minister Shri
Arun Jaitley has advocated the concept of Universal Basic Income (UBI).

This is as an alternative to the various social welfare schemes in an effort to reduce poverty.

The survey juxtaposes the benefits and costs of the UBI scheme.

The Survey says the UBI, based on the principles of universality, unconditionality and agency, is
a conceptually appealing idea.

Economic Survey points out that the districts where the needs are greatest are precisely the
ones where State capacity is the weakest.

This suggests that a more efficient way to help the poor would be to provide them resources
directly, through a UBI.

Exploring the principles and prerequisites for successful implementation of UBI, the Survey
points out that the two prerequisites for a successful UBI are:

a. functional JAM (Jan Dhan, Aadhar and Mobile) system as it ensures that the cash transfer
goes directly into the account of a beneficiary

b. Centre-State negotiations on cost sharing for the programme.

The Survey says that a UBI that reduces poverty to 0.5 percent would cost between 4-5 percent
of GDP.

This is assuming that those in the top 25 percent income bracket do not participate.

On the other hand, the existing middle class subsidies and food, petroleum and fertilizer
subsidies cost about 3 percent of GDP.

56. Answer: D- 6.75-7.25 percent

Explanation:
Survey was prepared by CEA in Finance Ministry, Arvind Subramaniam. The survey projects
growth rate of 6.75 to 7.25 percent in the fiscal 2017-2018.
This marks the transitional adverse impact of demonetisation on on GDP growth.

Highlights of Economic Survey 2016-2017


Growth Forecast:
Gross domestic product (GDP) growth in 2016-17 pegged at 6.5%, down from 7.6% in last fiscal
2015-16.

Like my face book page: India Bhai Page 16


100 Most Probable Questions for Prelims 2017(Current Affairs)

Economic growth to rebound to 6.75 to 7.5% in 2017-18.


Farm sector to grow at 4.1% in 2016-17, up from 1.2% in 2015-16.Growth rate of industrial
sector estimated to moderate to 5.2% in 2016-17 from 7.4% in 2015-16.
Service sector is estimated to grow at 8.9% in 2016-17 GST, other structural reforms should
take the growth rate trend to 8-10%.

Demonetisation:
The adverse impact of demonetisation on GDP growth will be transitional.
It will affect growth rate by 0.25-0.5%, but to have long-term benefits
It may affect supplies of certain agricultural products like sugar, milk, potatoes and onions.
Remonetisation will ensure that the cash squeeze is eliminated by April 2017.

Universal Basic Income (UBI):


Advocates the concept of UBI as an alternative to the various social welfare schemes in an
effort to reduce poverty.
It will be alternative to plethora of state subsidies for poverty alleviation. UBI would cost
between 4 and 5% of GDP

Taxation:
Prescribes cut in individual Income Tax rates, real estate stamp duties.
IT net could be widened gradually by encompassing all high income earners.
Time table for cutting corporate tax should be accelerated.
Tax administration could be improved to reduce discretion and improve accountability.
Goods and Services Tax (GST):
Fiscal gains from GST will take time to realise.

Fiscal Deficit:
Implementation of muted tax receipts, wage hike to put pressure on fiscal deficit in 2017-18.
For fiscal health of the economy fiscal prudence for both centre and states is needed.
Fiscal windfall from low oil prices to disappear in 2017-18.

Inflation:
The average consumer price index (CPI) inflation rate declined to 4.9% in 2015-16 from 5.9% in
2014-15.
CPI-based core inflation remained sticky around 5% in the 2016-17.
Oil prices, seen rising by one-sixth in 2017-18 over the previous fiscal 2016-17 prices which
could dampen India s economic growth.

Monetary Policy:
Monetary easing headroom may be capped due to sharp rise in prices in 2017-18.
Market interest rates seen lower in 2017-18 due to demonetisation.

Government Debt to GDP ratio:

Like my face book page: India Bhai Page 17


100 Most Probable Questions for Prelims 2017(Current Affairs)

It was 68.5% in 2016, down from 69.1% in 2015.

Banking:
Suggests setting up public sector asset rehabilitation agency (PSARA) to take charge of large
bad loans in banks.
With government backing, PSAR can overcome coordination and political issues on bad loans.

57. Answer: B: 4th

Explanation:
India has attained the fourth position globally in installed wind power capacity.
It is after China, US and Germany as a result of various steps in the right direction, the Economic
Survey said.
With the legal framework in place for the International Solar Alliance (ISA) by Prime Minister
Narendra Modi and launched during the UN climate summit in Paris, ISA will be a "major"
international body headquartered in India.
It said that currently, India's renewable energy sector is undergoing transformation with a
target of 175 GW of renewable energy capacity to be reached by 2022.

In order to achieve the target, the major programmes on implementation of :

Solar Park,
Solar Defence Scheme,
Solar scheme for Central Public Sector Undertakings,
Solar photovoltaic (SPV) power plants on canal bank and canal tops, solar pump, solar
rooftop among others have been launched.

A capacity addition of 14.30 GW of renewable energy has been reported during the last two
and half years under Grid Connected Renewable Power.This includes 5.8 GW from Solar Power,
7.04 GW from Wind Power, 0.53 GW from Small Hydro Power and 0.93 GW from Bio-power.
On October 31, 2016, India achieved 46.3 GW grid-interactive power capacity, 7.5 GW of grid-
connected power generation capacity in renewable energy. It also has small hydro power
capacity of 4.3 GW.
Besides this, 92,305 solar pumps were installed and Rs 38,000 crore worth of Green Energy
Corridor is being set up to ensure evacuation of renewable energy. With India's initiative, ISA
envisaged as a coalition of solar resource-rich countries to address their special energy needs,
will provide a platform to collaborate.It will focus on addressing the identified gaps through a
common and agreed approach.

ISA: Know More


24 countries have signed the Framework Agreement of ISA after it was opened for signature on
November 15, 2016.
ISA is expected to become inter-governmental treaty-based organization.

Like my face book page: India Bhai Page 18


100 Most Probable Questions for Prelims 2017(Current Affairs)

It will be registered under Article 102 of the UN charter after 15 countries ratify the Agreement.
With legal framework in place, ISA will be a major international body headquartered in India

58. Answer: D 1, 2 and 3


Explanation:
To tackle with the problems of increasing Non-Performing Assets (NPAs) of the banking system
and declining credit and investment, the Economic Survey 2016-17 on recommended a
centralised Public Sector Asset Rehabilitation Agency (PARA).

PARA will aim to look at the largest, most difficult cases, and make politically tough decisions to
reduce debt.

As per the Survey, gross NPAs have climbed to almost 12 per cent of gross advances for public
sector banks at end-September 2016.

At this level, India s NPA ratio is higher than any other major emerging market, with the
exception of Russia.

The consequent squeeze of banks has led them to slow credit growth to crucial sectors-
especially to industry and medium and small scale enterprises (MSMEs)-to levels unseen over
the past two decades.

As this has occurred, growth in private and overall investment has turned negative.

A decisive resolution is urgently needed before the Twin Balance Sheet problem becomes a
serious drag on growth. Public discussion of the bad loan problem has focused on bank capital.
A far more problematic issue is that of finding a way to resolve the bad debts.
Some debt repayment problems have been caused by diversion of funds.

But the vast majority has been caused by unexpected changes in the economic environment
after the Global Financial Crisis.
This caused timetables, exchange rates, and growth rate assumptions to go seriously wrong.
This concentration creates a challenge since large cases are difficult to resolve, but also an
opportunity.
The large debtors have many creditors, with different interests.
A professionally-run central agency with the government backing could overcome the
coordination and political issues that have impeded progress so far.

59. Answer: Big Data

Explanation:
For the first time, the Economic Survey has used Big Data Analysis to shed new light on the flow
of goods and people within India.

Like my face book page: India Bhai Page 19


100 Most Probable Questions for Prelims 2017(Current Affairs)

Survey produces first estimate of the flow of goods across states within India, based on
analyzing transactions level data provided by the Goods and Services Tax Network (GSTN),

Survey furnishes exciting new evidence on the flows of migrants within India.

This is based on detailed origin-destination passenger data provided by the Ministry of Railways
and on a new methodology for analysing the Census data.

This year s Economic Survey does not carry the usual statistical tables on the economy s
performance.

The survey seems to have compensated this by the use of Big Data and intensive data-mining of
multiple datasets.

The survey has used individual tax filings administered by the Goods and Service Tax Network
to estimate state-level (both inter and intra) trade.

Railway station-wise unreserved passenger traffic data provided by the Indian Railways has
been used to arrive at estimates of work-related migration.

Satellite imagery has been used to calculate built-up area and estimate potential property tax
collections (and hence losses being incurred).

Machine generated large scale data sets have been used more intensively. NSSO statistics has
been used to generate insights on spatial concentration of poverty and welfare beneficiaries.

Official statistical machinery has moved past surveys to include administrative data.

The new approach towards using diverse datasets is definitely an important first step towards
better decision-making.

Marrying satellite imagery about properties data with something like income tax data for
India s top 50 cities and house-size census data can generate rich insights about Indian cities.

60. Answer: A: Females

Explanation:
New estimates of labour migration in India have revealed that inter-state labor mobility is
significantly higher than previous estimates.

This was stated in the Economic Survey 2016-17.


The study based on the analyses of new data sources and new methodologies also shows that
the migration is accelerating and was particularly pronounced for females.
The data sources used for the study are the 2011 Census and railway passenger traffic flows of

Like my face book page: India Bhai Page 20


100 Most Probable Questions for Prelims 2017(Current Affairs)

the Ministry of Railways and new methodologies including the Cohort-based Migration Metric
(CMM) .

The new Cohort-based Migration Metric (CMM) shows that inter-state labor mobility averaged
5-6.5 million people between 2001 and 2011.

This is yielding an inter-state migrant population of about 60 million and an inter-district


migration as high as 80 million.

The first-ever estimates of internal work-related migration using railways data for the period
2011-2016 indicate an annual average flow of close to 9 million migrant people between the
states.

Both these estimates are significantly greater than the annual average flow of about 4 million
suggested by successive Censuses and higher than previously estimated by any study.

Migration for work and education is also accelerating.

There is also a doubling of the stock of inter-state out migrants to nearly 12 million in the 20-
29year old cohort alone.

Higher growth and a multitude of economic opportunities could therefore have been the
catalyst for such an acceleration of migration.

Language does not seem to be a demonstrable barrier to the flow of people.

Fourth, the patterns of flows of migrants found in this study are broadly consistent with what is
expected - less affluent states see more out migration migrating out while the most affluent
states are the largest recipients of migrants.

Policy actions to sustain and maximize the benefits of migration include:

Ensuring portability of food security benefits,


Providing healthcare and a basic social security framework for migrants - potentially
through an inter-state self-registration process.

Redistributive Resource Transfer and the Economic Survey

Economic Survey also calculates Redistributive Resource Transfers (RRT) from the Centre
(between 1994 and 2015) and value of natural resources for Indian States (over 1980 and
2014).

It correlates these with several economic outcomes and an index of governance.

Like my face book page: India Bhai Page 21


100 Most Probable Questions for Prelims 2017(Current Affairs)

Redistributive Resource Transfer or RRT to a state (from the Centre) is defined as gross
devolution to the state adjusted for the respective state s share in aggregate Gross Domestic
Product(GDP).

The top 10 recipients are:

Sikkim,
Arunachal Pradesh,
Mizoram,
Nagaland,
Manipur,
Meghalaya,
Tripura,
Jammu and Kashmir
Himachal Pradesh
Assam.

61. Answer: A: States must take consent of the Centre for additional borrowing
62. Answer: C: All are correct

63. Answer: A: Overleveraged companies and bad-loan-encumbered banks

Explanation:
The Economic Survey of 2015-16 acknowledges that one of the critical challenges confronting
the Indian economy is the twin balance sheet problem. The balance sheets of both public
sector banks (PSBs) and some corporate houses are in terrible shape and it has been seen as a
major obstacle to investment and reviving growth.

The problems faced by the Public Sector Banks are linked directly to that of the corporate
sector. During the boom years, some companies borrowed a lot of money from banks to invest
in infrastructure and commodity- related businesses, such as steel, power, infrastructure etc.
But now, due to slump in both these sectors, the corporate profits have hit new lows. With low
profits, the corporates are not able to repay their loans and their debts are rising at an
alarming level. They have no other option other than to cut back investments.

Let us understand some basic terms to appreciate the full import of the troubles faced by the
economy.

What are Non-Performing Assets?

The NPAs are assets that stop generating income for a bank. Bank s assets mostly comprise of
loans and when these loans are on the verge of default (that is, about to go bad), they are
classified as NPA.

Like my face book page: India Bhai Page 22


100 Most Probable Questions for Prelims 2017(Current Affairs)

In India, a loan is classified as NPA, if the interest or any installment remains unpaid for a period
of more than 90 days.

The gross NPAs in India were 5.1 % of total loans advanced by the public sector banks as of
September 2015 and the stressed assets were 11% of total loans advanced by them.

What are stressed assets?

Stressed assets are NPAs plus restructured assets. Restructured loans are loans that have been
converted to equity under the corporate debt restructuring scheme.

The high amount of NPAs in banks have hit their profitability as well, as banks have to make
more provisioning. Banks have to set aside large funds as provisions to take care of the
potential losses arising out of the loans that might go bad. NPAs are the reason why most banks
reported losses in the last quarter.

Banks NPAs have been growing for a while now. It not only affects the balance sheet and
profitability of banks but also limits credit availability to the corporate sector. This limited credit
availability leads to further decline in private investment,

The Union budget of 2016 has allocated Rs 25000 crore towards recapitalisation of Public
Sector banks. This is a necessary step to infuse capital into the Public Sector Banks. RBI has set
March 2017 as the deadline for banks to clean up their balance sheets and strengthen their
assets and has taken several measures to address the problem.

64. Answer: C: All are correct


65. Answer: B: Only 2
Explanation:
On the financial front, there has been divergence on both income and convergence

66. Answer: C: Both 1 and 2


Explanation:
What is RRT?
RRT to a state is defined as gross devolution3 to the state adjusted for the respective state s
share in aggregate gross domestic product. Thus RRT is not identical to gross devolution.
The higher the RRT means
The slower is growth of state.
The smaller is the share of manufacturing in GSDP.
The lower is own tax revenues.
The economic survey pointed out that there is no evidence of a positive relationship between
these transfer and economic outcome including per capita consumption, GSDP growth,
institutional quality.
Why resource abundance state grow less rapidly than resource less state?
It is due to resource curse which is attributed due to following

Like my face book page: India Bhai Page 23


100 Most Probable Questions for Prelims 2017(Current Affairs)

First, the exploitation of natural resources generates rents, which lead to rapacious rent-
seeking (the voracity effect) and increased corruption.
Second, natural resource ownership exposes countries to commodity price volatility, which can
destabilize GDP growth.
Finally, natural resource ownership like foreign aid makes countries susceptible to Dutch
disease .
Way ahead for development
Infrastructure and Connectivity:
RRT curse and natural resource curse , could be a result of poor connectivity in
particular and poor infrastructure physical, financial, and digital in general.
This is clearly true of the north-east but also true of many parts of resource-rich India.
Enhancing connectivity financial and physical on a war footing (as the government
has attempted for financial inclusion with the Pradhan Mantri Jan Dhan Yojana
(PMJDY), expediting the optical fibre network, etc will have a moderating effect.

Redistributive Resource Transfers


In a federal system the Centre must play a redistributive role: it will always have to redirect
resources to under-developed states. Rather, the Centre will need to find ways of ensuring that
the resources it redistributes are used more productively.
Redirecting flows to households
One possibility would be to redirect a certain portion of RRT and channel the resources directly
to households as part of a Universal Basic Income (UBI) scheme.
Conditioning transfers on fiscal performance
Finance Commissions could revert to the practice of the 13th FC of conditioning transfers on
the tax effort of states .This will offset the fiscal bias.
Making governance- contingent transfers
To encourage better governance and sound institutional practices, the fund transfer
mechanism could explicitly include a few monitorable institutional indicators as criteria for
receiving transfers.

Redistributive Resource Transfers maybe defined as the architecture of redistributive resource


flows from the Centre to the States as a part of Devolution of Funds, which play a key role in
bridging inter-States inequality and as aid to under-developed states over-and-above their
contribution to GDP.

The key differences between RRT and Foreign Aid are:

1.
~Foreign Aid adds to the National Disposable Income of the economy. Therefore there is a risk
of economic dependence to the foreign nation. There is also possibility of economic collapse in
case of sudden changes in the Aid structure and foreign relations, impacting sovereignty. Eg.-
Economic collapse in Madagascar in 2009 after withdrawal of UK Foreign Aid due to political
turmoil.

Like my face book page: India Bhai Page 24


100 Most Probable Questions for Prelims 2017(Current Affairs)

~RRTs are national redistributions and therefore donot augment the National Disposable
Income. This allows for a more controlled economic structure.

2.
~Foreign Aid invariably results in a Donor-Recipient relationship. Since institutional
development of tax base and economic management are more critical than simple resource
availability, such a relationship may be regressive. Aid dependent nations have been shown to
have low incentive to develop a domestic revenue regime.

~RRTs on the other hand, are part of a national structure and may be controlled by the Centre
to incentivize and spur development in States. For example, - the mandate of 13th Finance
Commission that conditioned RRT aids on the tax efforts of respective States.

Therefore, we may conclude that RRTs are an element of Welfare nature of Indian polity that
commits itself to ensuring equitable development across states by providing fiscal stimulus to
less developed states as an incentive to develop a robust revenue regime. However, as per
Economic Survey 2017, the ills of foreign aid may manifest in RRT structure as well unless
carefully monitored from the Centre.
67. Answer: D
68. Answer: A
Explanation:
As the debate around Demonetisation rages on in the country, the economists have expressed
varied opinions on this unprecedented reform. Here is the long term and short term impact of
government's move to make India a cashless economy.
Short-term impact Long-term Impact
With a tax net widening after restriction
GDP GDP growth to be negatively impacted
on cash economy, GDP in the long-term
Growth by 0.5-1% due to drop in consumption
likely to get a boost
Since effect of lower demand can have a
Inflation likely to come down due to low
Inflation lagging impact on inflation, prices may
demand owing to liquidity problem
remain flat or fall in the long term
If Demonetisation boots formal banking
Interest With Banks flush with cash, interest
and more cash continues to come in the
rates rates may come down in the short-term
system, interest rates may fall further
Yields likely to fall as liquidity in the Likely to remain flat to positive
G-sec yield banking system means interest rates are depending on how inflation plays out
likely to fall and how's the country's fiscal situation
Unlikely to have much impact as most of
With likely increase in the size of formal
Fiscal the gains (RBI surplus, higher tax
economy, tax collections would improve
Deficit collections, penalties) will accrue after
improving the country's fiscal situation.
FY2017.

Like my face book page: India Bhai Page 25


100 Most Probable Questions for Prelims 2017(Current Affairs)

69. Answer: B: 1,3,4 & 5 only


Explanation:
See table in answer 68.

70. Answer: A: 1 & 3 only


Explanation:
To increase the share of non-fossil fuels based power generation capacity to 40 per cent of
installed electric power capacity by 2030.

71. Answer: D: 1, 2, 3, 4, and 5

72. Answer: A: 1 2 3

73. Answer: B: 2 3 1

74. Answer: D: 1, 2, 3 and 4

75. Answer: D: 1, 2, 3 and 4

76. Answer: A: Only 1

77. Answer: A: 1, 3, 2

78. Answer: A: Only 1

79. Answer: D: TB by 2025

80. Answer: D

81. Answer: D

82. Answer: B

83. ANSWER: A: Only 1


Explanation:
The Fund will be managed by a trust headed by Finance Minister. The Fund will have the
objective of contributing to the national effort of cleaning of river Ganga.
84. ANSWER: C: Both 1 and 2

Like my face book page: India Bhai Page 26


100 Most Probable Questions for Prelims 2017(Current Affairs)

Explanation:
NMCG is the implementation wing of the NGRBA. NGRBA can extend area for effective
abatement of pollution of the river.
85. ANSWER: A: Only 1 and 4
Explanation:
Swachha Bharat Abhiyaan and National Rural Livelihood Mission (NRLM) are implemented with
changed sharing pattern by centre and the states.
86. Answer: B: For information on eligibility of benefits under various social sector schemes
Explanation:
It has been implemented as a common man s interface for providing comprehensive
information on eligibility of benefits under various social sector schemes operational across the
country. The information is provided through Web based portal, Call Centre and Interactive
Voice Response System (IVRS).
87. Answer: B : Special Scholarship Scheme for North East Region (NER)
Explanation:
The University Grants Commission (UGC) has launched Ishan Uday a Special Scholarship
Scheme for North East Region (NER) from academic session 2014-15. Under the scheme ten
thousand fresh scholarships are to be provided for general degree course, technical and
professional courses including medical and Para-medical courses.
88. Answer: D: 1, 2 and 3
Explanation:
The Mission will be implemented with the objectives to: a) development and conservation of
indigenous breeds b) undertake breed improvement programme for indigenous cattle breeds
so as to improve the genetic makeup and increase the stock; c) enhance milk production and
productivity; d) upgrade nondescript cattle using elite indigenous breeds like Gir, Sahiwal,
Rathi, Deoni, Tharparkar, Red Sindhi and e) distribute disease free high genetic merit bulls for
natural service.
89. ANSWER: B: Only 1, 3 and 5
Explanation:
Anti infiltration duties are peace time duties and not war time. Other war time duties are
maintenance of law and order in enemy territory under army s control; assistance in control of
refugees; performing special tasks connected with intelligence including raids; Aggression

Like my face book page: India Bhai Page 27


100 Most Probable Questions for Prelims 2017(Current Affairs)

against enemy army is not allowed for BSF. Limited aggression against central police or irregular
forces of enemy is allowed.
90. Answer: B: Only 2 and 3
Explanation:
1st phase states were Bihar, Chhattisgarh, Jharkhand, MP, UP, Odisha and Rajasthan. 2nd will
focus efforts in 1-2 selected districts in the 8 states of Bihar, Chhattisgarh, Jharkhand,
Maharashtra, MP, UP, Odisha and Rajasthan. The scheme has also been extended for 5 years to
NE and J&K.

91. Answer: D: Upgrading the skills and hygiene standards of Street Food Vendors
Explanation:
Swachh Bharat Swachh Pakwan (Hunar Zaika): The street food vendors constitute a significant
percentage of the hospitality service providers, are part of the Indian milieu through the ages
and have a pan India presence. This program is aimed at upgrading the skills and hygiene
standards of Street Food Vendors, so that they become a distinctive aspect of the Indian
tourism. The Ministry of Tourism is partnering with the National Association of Street Vendors
of India (NASVI) for the specific purpose of orientation, skill testing and certification of the
vendors. The NASVI will either directly or through its city co-coordinators nominate vendors in
groups for training at one of the Institute of Hotel Management (IHMs) / Food Craft Institute
(FCIs) authorized by the MoT to conduct such programmes. Since the vendors will also be
sensitized, as part of the orientation, to personal, environmental and waste disposal hygiene,
this MoT s effort in partnership with NASVI will also visibly contribute to the Swachch Bharat
Abhiyan.

92. Answer: B: Ministry of Tourism


Explanation:
The Ministry of Tourism launched a special initiative called Hunar Se Rozgar Tak (HSRT) in the
year 2009-10 for creation of employable skills amongst youth. The initiative is fully funded by
the Ministry of Tourism. It covers trades like food production, food & beverage service,
housekeeping Utility, Bakery & Patisserie, Driving, Stone Masonry, Golf Caddies and Tourist
Facilitation, etc. In December, 2014, the HSRT Initiative was opened to private participation
under a distinct vertical titled Hunar Se Rozgar Tak: Badhate Kadam. The objective is to enhance

Like my face book page: India Bhai Page 28


100 Most Probable Questions for Prelims 2017(Current Affairs)

the programme s reach and delivery. Now the Industrial units, associations of industries,
professional/ skill developing agencies with proven credentials; and Hospitality Institutes
approved by the AICTE / National Skill Development Authority / State Governments / Union
Territory Administrations can participate and contribute to the skill needs in the hospitality
trade. The trades opened are: food production, food & beverage service, house-keeping and
bakery & patisserie.

93. Answer: A: Only 1 and 2


Explanation:
5 Circuits are identified initially North-East Circuit, Buddhist Circuit, Coastal Circuit, Himalayan
Circuit and Krishna Circuit. 3 projects one each in North-East, Buddhist and Coastal Circuits are
sanctioned in 2014-15. The Odisha Government has sought inclusion of Odisha s Diamond
Triangle of Buddhist sites (Ratnagiri, Udayagiri, and Lalitagiri) in the SWADESH DARSHAN
Scheme. Tourism Ministry has already identified Bodh Gaya, Rajgir and Vaishali in Bihar and
Kapilavastu, Sarnath, Sravasti and Kushinagar in Uttar Pradesh under the Buddhist circuit of
Swadesh Darshan. Even though the Diamond Triangle in Odisha has immense potential to draw
domestic and international travellers, the Union Ministry is yet to shift its focus towards it due
to lack of tourist infrastructure at these sites. Funds from Swadesh Darshan can help in
developing tourism amenities at these sites.

94. Answer: C: Domestic demand


Explanation:
In India although the growth is driven by domestic demand, contribution of exports to
economic growth Is important as exports of goods and services account for about one fourth of
the GDP. Domestic consumption is more and is fuelling the current growth

95. Answer: B
Explanation: strengthening border protection.
The Committee constituted under the Chairmanship of Shri Madhukar Gupta, has submitted its
report to the Government to strengthen border protection and address vulnerabilities in
fencing along the Indo- Pakistan border. The Committee has given its recommendations broadly
on the issues of Threats and Border Protection, assessment of force level, deployment on the
border, infrastructure and technology issues for protection of border and administrative issues.
96. ANSWER: A: Aadhaar-enabled smart cards

Like my face book page: India Bhai Page 29


100 Most Probable Questions for Prelims 2017(Current Affairs)

Explanation:
The recent Budget development on Aadhaar is that senior citizens would receive Aadhaar-
based smart cards for their health and well-being.
Aadhaar-based smart cards would comprise health details of senior citizens.
The FM further said that the pilot service of these cards would take off in 15 cities initially in
India beginning this year.
The announcement for this new initiative shows how the government is focused on the path of
making Aadhaar the core platform in different services.
The government recently said that as of now more than 111 crore citizens in the country have
an Aadhaar number. This unit covers more than 99 percent of the Indian adult population.
Following demonetization, the enrollment for Aadhaar also increased to 7 to 8 lakh per day as
against 5 to 6 lakh till October 2016.
The government also reported a rise of 2.69 crore transactions of Aadhaar-based enabled
payments in November 2016, which increased to 3.73 crore in December 2016 and 2.06 crore
transactions in the first half of January.

97. Answer: D: Only A and B


Explanation:
First railway budget in 93 years to be announced as part of the Union budget announced the
largest ever allocation of INR. 1.3 trillion to Indian Railways, with a gross budgetary support of
INR. 55,000 crore.
National carrier will focus on 4 major areas: passenger safety, capital and development works,
cleanliness and finance and accounting reformsmatters traditionally announced by the Union
railway minister.

Highlights
A corpus of INR 1 trillion for a rail safety fund to be spent over five years; solar power for 7,000
railway stations;
Redevelopment of 25 railway stations;
70 projects for construction and development through joint ventures with nine state
governments;

Like my face book page: India Bhai Page 30


100 Most Probable Questions for Prelims 2017(Current Affairs)

Commissioning of 3,500km railway lines in 2017-2018, up from 2,800 km in 2016-17.

The budget also proposed stock market listing of railway enterprises like Indian Railways
Catering and Tourism Corporation (IRCTC), Ircon International Ltd and Indian Railways Finance
Corporation (IRFC), and end-to-end transport solutions by Indian Railways in selected
commodities by partnering logistics companies.
The budget also waived service charges on railway e-tickets to encourage cashless transactions
and made AC class tickets cheaper by INR 40 and sleeper class by INR 20.
The solar power which will help Indian Railways accomplish its Mission 41K, a plan to save INR
41,000 crore in next 10 years by electrifying 90 percent of railway lines, and also reduce
expenditure on power was announced

Railways plans to develop 1,000 mega-watt (MW) solar (power) at 7,000 stations creates
distributed solar ecosystem across India.
100 percent electrification of rural areas should also integrate solar distributed generation as
part of the scheme for better reliability.
Freight loading is expected at 1,165 million tonnes (MT) in 2017-18, which is 71.50MT over
revised estimates 2016-17 and earnings at Rs1,18,998 crore.
Railways is expected to earn Rs50,125 crore in passenger traffic, taking total receipts to
Rs1,89,498.37 crore.

99. Answer: A: Only 1


Explanation:
An EVM consists of a control unit and a balloting unit. The control unit is with the
Election Commission appointed polling officer; the balloting unit is in the voting
compartment into which the voter enters to cast her vote in secret by pressing the
button against the name and symbol of the candidate of her choice. The machine is
electronically protected to prevent any tampering/manipulation. The programme
(software) used in these machines is burnt into a One Time Programmable
(OTP)/Masked chip so that it cannot be altered or tampered with. Hence, statement 1 is
correct.
ECI uses Voter Verifiable Paper Audit Trail (VVPAT), which gives a verified paper record
with a view to enhance transparency. Further these machines are not networked either
by wire or by wireless to any other machine or system. Therefore, there is no possibility
of its data corruption. Hence, statement 2 is not correct.

Like my face book page: India Bhai Page 31


100 Most Probable Questions for Prelims 2017(Current Affairs)

100. Answer: D: 1, 2, 3, 4 and 5

Explanation:

Coalition for Epidemic Preparedness Innovations


The Coalition for Epidemic Preparedness Innovations (CEPI) has officially launched at 2017
World Economic Forum (WEF) to create new vaccines for emerging infectious diseases.

The CEPI has been formed with initial funding of $460 million from the governments of
Germany, Japan and Norway, as well as the Bill & Melinda Gates Foundation and the
Wellcome Trust.

Initially, the CEPI group is targeting the 3 infectious diseases viz. Middle East respiratory
syndrome (MERS)-coronavirus, Lassa fever, and Nipah virus. All three have the potential to
cause large outbreaks and the goal is to have two potential vaccines for each of these viruses
ready before an epidemic hits. The group may also provide support for research on strains of
the Ebola, Marburg and Zika viruses.

--------------------------------------------

Like my face book page: India Bhai Page 32

You might also like